N208 NCLEX Questions

अब Quizwiz के साथ अपने होमवर्क और परीक्षाओं को एस करें!

Which factor is an indicator for a diagnosis of hydronephrosis? A. History of nocturia B. History of urinary stones C. Recent weight loss D. Urinary incontinence

B. Causes of hydronephrosis or hydroureter include tumors, stones, trauma, structural defects, and fibrosis.

When assessing a client with pyelonephritis, the nurse recognizes that which of these conditions may predispose the client to the problem? A. Spinal cord injury B. Cardiomyopathy C. Hepatic failure D.Glomerulonephritis

A. Chronic pyelonephritis occurs with spinal cord injury, bladder tumor, prostate enlargement, or urinary tract stones.

The client in shock has the following vital signs: T 99.8° F, P 132 beats/min, R 32 breaths/min, and BP 80/58 mm Hg. Calculate the pulse pressure.

22 mm Hg Pulse pressure is the difference between the systolic and diastolic pressures: 80 (systolic) - 58 (diastolic) = 22 (pulse pressure)

What are serious side effects of antiviral agents prescribed for a client with acute myelogenous leukemia? (Select all that apply.) A. Cardiomyopathy B. Nephrotoxicity C. Ototoxicity D. Stroke E. Diarrhea

B, C Antiviral agents, although helpful in combating severe infection, have serious side effects, especially nephrotoxicity and ototoxicity. Cardiomyopathy and stroke are not serious side effects of antiviral agents. Diarrhea is a mild side effect associated with antibiotic therapy.

The charge nurse is making client assignments for the day shift. Which client would be best to assign to an LPN/LVN? A. A client who has just returned from having a kidney artery angioplasty B. A client with polycystic kidney disease who is having a kidney ultrasound C. A client who is going for a cystoscopy and cystourethroscopy D. A client with glomerulonephritis who is having a kidney biopsy

B. Kidney ultrasounds are noninvasive procedures without complications; the LPN/LVN can provide this care.

What are the risk factors for the development of leukemia? (Select all that apply.) A. Bone marrow hypoplasia B. Chemical exposure C. Down syndrome D. Ionizing radiation E. Multiple blood transfusions F. Prematurity at birth

A, B, C, D Reduced production of blood cells in the bone marrow is one of the risk factors for developing leukemia. Exposure to chemicals through medical need or by environmental events can also contribute. Certain genetic factors contribute to the development of leukemia; Down syndrome is one such condition. Radiation therapy for cancer or other exposure to radiation, perhaps through the environment, also contributes. There is no indication that multiple blood transfusions are connected to clients who have leukemia. Although some genetic factors may influence the incidence of leukemia, prematurity at birth is not one of them.

A client has a diagnosis of lung cancer. To which areas does the nurse anticipate that this client's tumor may metastasize? (Select all that apply.) A. Brain B. Bone C. Lymph nodes D. Kidneys E. Liver

A, B, C, E Typical sites of metastasis of lung cancer include the brain, bone, liver, lymph nodes, and pancreas. Kidneys are not a typical site of lung cancer metastasis.

A distant family member arrives to visit a female client recently diagnosed with leukemia. The family member asks the nurse, "What should I say to her?" Which responses does the nurse suggest? (Select all that apply.) A. "Ask her how she is feeling." B. "Ask her if she needs anything." C. "Tell her to be brave and to not cry." D. "Talk to her as you normally would when you haven't seen her for a long time." E. "Tell her what you know about leukemia."

A, B, D Asking the client how she is feeling is a broad general opening and would be nonthreatening to the client. Asking if she needs anything is a therapeutic communication of offering self and would be considered to be therapeutic and helpful to the client. The family member should talk to her as she normally would when she hasn't seen her in a long time. There is no need to act differently with the client. If she wants to offer her feelings, keeping a normal atmosphere facilitates that option. Acting as if things are "different" because she has cancer takes the control of the situation from the client. Telling her to be brave and not to cry is callous and unfeeling; if the client is feeling vulnerable and depressed, telling her to "be brave" shuts off any opportunity for her to express her feelings. There is no need to inform the client about her disease, unless she asks about it. Opening the conversation with discussion about leukemia should be the client's prerogative.

The nurse includes which factors in teaching regarding the typical warning signs of cancer? (Select all that apply.) A. Persistent constipation B. Scab present for 6 months C. Curdlike vaginal discharge D. Axillary swelling E. Headache

A, B, D Change in bowel habits, a sore that does not heal, and a lump or thickening in the breast or elsewhere are all potential warning signs of cancer. Curdlike vaginal discharge represents a yeast infection. Headache is not a warning sign, but may be present with multiple problems.

The nurse is teaching a client with newly diagnosed anemia about conserving energy. What does the nurse tell the client? (Select all that apply.) A. "Allow others to perform your care during periods of extreme fatigue." B. "Drink small quantities of protein shakes and nutritional supplements daily." C. "Perform a complete bath daily to reduce your chance of getting an infection." D. "Provide yourself with four to six small, easy-to-eat meals daily." E. "Perform your care activities in groups to conserve your energy." F. "Stop activity when shortness of breath or palpitations are present."

A, B, D, F It is critical to have others help the anemic client who is extremely tired. Although it may be difficult for him or her to ask for help, this practice should be stressed to the client. Drinking small protein or nutritional supplements will help rebuild the client's nutritional status. Having four to six small meals daily is preferred over three large meals; this practice conserves the body's expenditure of energy used in digestion and assimilation of nutrients. Stopping activities when strain on the cardiac or respiratory system is noted is critical. A complete bath should be performed only every other day; on days in between, the client can be taught to take a "mini" sponge bath, which will conserve energy and still be safe in preventing the risks for infection. Care activities should be spaced every hour or so rather than in groups to conserve energy; the time just before and after meals should be avoided.

When monitoring a client with suspected syndrome of inappropriate antidiuretic hormone (SIADH), the nurse reviews the client's medical record, which contains the following information. The nurse notifies the health care provider for which signs and symptoms consistent with this syndrome? (Select all that apply.) Neuro: Episodes of confusion Cardiac: Pulse 88 and regular Musculoskeletal: Weakness, tremor Na: 115, K: 4.2, Creatinine: 0.8 ondansetron (Zofran) cyclophosphamide (Cytoxan) A. Hyponatremia B. Mental status changes C. Azotemia D. Bradycardia E. Weakness

A, B, E Antidiuretic hormone (ADH) is secreted or produced ectopically, resulting in water retention and sodium dilution. Dilutional hyponatremia results from ADH secretion, causing confusion and changes in mental status. Weakness results from hyponatremia. Azotemia refers to buildup of nitrogenous waste products in the blood, typically from renal damage. Bradycardia is not part of the constellation of symptoms related to SIADH; tachycardia may result from fluid volume excess.

When caring for the client receiving cancer chemotherapy, which signs or symptoms related to thrombocytopenia should the nurse report to the health care provider? (Select all that apply.) A. Bruises B. Fever C. Petechiae D. Epistaxis E. Pallor

A, C, D Bruising, petechiae, and epistaxis (nosebleeds) are symptoms of a low platelet count. Fever is a sign of infection secondary to neutropenia. Pallor is a sign of anemia.

Which potential side effects does the nurse include in the teaching plan for a client undergoing radiation therapy for laryngeal cancer? (Select all that apply.) A. Fatigue B. Changes in color of hair C. Change in taste D. Changes in skin of the neck E. Difficulty swallowing

A, C, D, E Radiation therapy to any site produces fatigue in most clients, and may cause clients to report changes in taste. Radiation side effects are site-specific; the larynx is in the neck, so changes in the skin of the neck may occur. Dysphagia (difficulty swallowing) may occur from radiation to the throat area. Chemotherapy, which causes alopecia, may cause changes in the color or texture of hair; this does not normally occur with radiation therapy.

The older adult woman who reports a change in bladder function says, "I feel like a child who sometimes pees her pants." What is the nurse's best response? A. "Have you tried using the toilet at least every couple of hours?" B. "How does that make you feel?" C. "We can fix that." D. "That happens when we get older."

A. By emptying the bladder on a regular basis, urinary incontinence from overflow may be avoided, which may give the client some sense of control.

The nurse has received in report that a client receiving chemotherapy has severe neutropenia. Which interventions does the nurse plan to implement? (Select all that apply.) A. Assess for fever. B. Observe for bleeding. C. Administer pegfilgrastim (Neulasta) - a biologic response modifier D. Do not permit fresh flowers or plants in the room. E. Do not allow the client's 16-year-old son to visit. F. Teach the client to omit raw fruits and vegetables from the diet.

A, C, D, F Any temperature elevation in a client with neutropenia is considered a sign of infection and should be reported immediately to the health care provider. Administration of biological response modifiers, such as filgrastim (Neupogen) and pegfilgrastim (Neulasta), is indicated in neutropenia to prevent infection and sepsis. Flowers and plants may harbor organisms such as fungi or viruses and are to be avoided for the immune-suppressed client. All fruits and vegetables should be cooked well; raw fruits and vegetables may harbor organisms. Thrombocytopenia, or low platelet levels, causes bleeding, not low neutrophils (a type of white blood cell). The client is at risk for infection, not the visitors, if they are well; however, very small children, who may get frequent colds and viral infections, may pose a risk.

A 32-year-old female with a urinary tract infection reports urinary frequency, urgency, and some discomfort upon urination. Her vital signs are stable except for a temperature of 100° F. What information does a nurse provide about taking her prescribed Bactrim? Select all that apply. A. "Be certain to wear sunscreen and protective clothing." B. "Drink at least 3 liters of fluids every day." C. "Take this drug with 8 ounces of water." D. "Try to urinate frequently to keep your bladder empty." E. "You will need to take all of this drug to get the benefits."

A,B,C,E: Wearing sunscreen and protective clothing is important to do while on this drug. Increased sensitivity to the sun can lead to severe sunburn. Sulfamethoxazole can form crystals that precipitate in the kidney tubules. Fluid intake prevents this complication. Clients should be cautioned to take all of the drug that is prescribed for them, even if their symptoms improve or disappear soon. Emptying the bladder is important-but not keeping it empty-as is stated here. The client should be advised to urinate every 3 to 4 hours or More often if he or she feels the urge

Which client with a long-term urinary problem does the nurse refer to community resources and support groups? Select all that apply. A. 32-year-old with a cystectomy B. 44-year-old with a Kock pouch C. 48-year-old with urinary calculi D. 78-year-old with urinary incontinence E. 80-year-old with dementia

A,B,D: The client with a cystectomy and a Kock pouch, and a urinary incontinence would benefit from community resources and support groups. Others who have had their bladders removed are good sources for information and for help in establishing coping mechanisms.

The client is scheduled for intravenous urography. During the assessment, the nurse notes a previous reaction of urticaria, itching, and sneezing to contrast dye. Which precautions does the nurse take? Select all that apply. A. Ensures that an antihistamine and a steroid are prescribed B. Documents the reaction on the chart C. Uses no contrast dye for the procedure D. Cancels the procedure E. Ensures that the health care provider is aware of the reaction

A,B,E: Suppression of immune and allergic responses should be undertaken. Allergies and suspected allergies must be documented in the medical record. The nurse must notify the provider of the previous response and obtain requests for antihistamine and corticosteroid. Contrast will give a more clear picture; if the provider believes it is necessary, suppression of the immune response is an acceptable intervention. The scope of practice for professional nursing does not include prescribing or canceling procedures.

Which client with an indwelling urinary catheter does a nurse re-assess to determine whether the catheterization needs to be continued or can be discontinued? Select all that apply. A. Three-day postoperative client B. Client in the step-down unit C. Comatose client with careful monitoring I/Os D. Incontinent client with perineal skin breakdown E. Incontinent long-term care older adult

A,B,E: Three days after surgery, the client probably should be able to urinate on his or her own. If the surgery was a bladder or urethral repair, then discontinuing the catheter might not be a consideration so soon. However, most clients do not need long-term catheterization after they have surgery. The incidence of complications (colonization of bacteria) begins to increase after 48 hours postinsertion.The client who is out of an intensive care situation is definitely one who should be considered for discontinuation of his or her catheter. He or she should be somewhat ambulatory and able to get to a bedside commode.The comatose client who is on strict I&O needs to have a urinary catheter in place to keep accurate account of his or her fluid balance.

A nurse is teaching a group of older adult women about the signs and symptoms of urinary tract infection (UTI). Which concepts does the nurse explain in the presentation? Select all that apply. A. Dysuria B. Enuresis C. Frequency D. Nocturia E. Urgency F. Polyuria

A,C,D, E: Dysuria, Frequency, Nocturia and urgency are symptoms of a UTI. Enuresis-bed-wetting and polyuria-are not signs of a UTI.

The nurse is teaching a client about induction therapy for acute leukemia. Which client statement indicates a need for additional education? A. "After this therapy, I will not need to have any more." B. "I will need to avoid people with a cold or flu." C. "I will probably lose my hair during this therapy." D. "The goal of this therapy is to put me in remission."

A. "After this therapy, I will not need to have any more." Induction therapy is not a cure for leukemia, it is a treatment; therefore, the client needs more education to understand this. Because of infection risk, clients with leukemia should avoid people with a cold or flu. Induction therapy will most likely cause the client with leukemia to lose his or her hair. The goal of induction therapy is to force leukemia into remission.

When teaching fire safety to parents at a school function, which advice does the school nurse offer about the placement of smoke and carbon monoxide detectors? A. "Every bedroom should have a separate smoke detector." B. "Every room in the house should have a smoke detector." C. "If you have a smoke detector, you don't need a carbon monoxide detector." D. "The kitchen and the bedrooms are the only rooms that need smoke detectors."

A. "Every bedroom should have a separate smoke detector." Teach all people to use home smoke detectors and carbon monoxide detectors and to ensure these are in good working order. The number of detectors needed depends on the size of the home. Recommendations are that each bedroom has a separate smoke detector, there should be at least one detector in the hallway of each story, and at least one detector is needed for the kitchen, each stairwell, and each home entrance. Carbon monoxide detectors are instrumental in picking up other types of carbon monoxide gas, such as from a defective heating unit.

The nurse is teaching a 47-year-old woman about recommended screening practices for breast cancer. Which statement by the client indicates understanding of the nurse's instructions? A. "My mother and grandmother had breast cancer, so I am at risk." B. "I get a mammogram every 2 years since I turned 30." C. "A clinical breast examination is performed every month since I turned 40." D. "A computed tomography (CT) scan will be done every year after I turn 50."

A. "My mother and grandmother had breast cancer, so I am at risk." A strong family history of breast cancer indicates a risk for breast cancer. Annual rather than biannual screening may be indicated for a strong family history. An annual mammogram is performed after age 40 or in younger clients with a strong family history. The client may perform a self-breast examination monthly; a clinical examination by a health care provider is indicated annually. Annual CT breast scans after age 50 are not a current recommendation.

Which information must the organ transplant nurse emphasize before a client is discharged? A. "Taking immunosuppressant medications increases your risk for cancer and the need for screenings." B. "You are at increased risk for cancer when you reach 60 years of age." C. "Immunosuppressant medications will decrease your risk for developing cancers." D. "After 6 months, you may stop immunosuppressant medications, and your risk for cancer will be the same as that of the general population."

A. "Taking immunosuppressant medications increases your risk for cancer and the need for screenings." Use of immunosuppressant medications to prevent organ rejection increases the risk for cancer. Advanced age is a risk factor for all people, not just for organ transplant recipients. Immunosuppressant medications must be taken for the life of the organ; the risk for developing cancer remains.

What is the best method to prevent autocontamination for a client with burns? A. Change gloves when handling wounds on different areas of the body. B. Ensure that the client is in isolation therapy. C. Restrict visitors. D. Watch for early signs of infection.

A. Change gloves when handling wounds on different areas of the body. Gloves should be changed when wounds on different areas of the body are handled and between handling old and new dressings. Isolation therapy methods and restricting visitors are used to prevent cross-contamination, not autocontamination. Watching for early signs of infection does not prevent contamination.

Which client is at greatest risk for having a hemolytic transfusion reaction? A. A 34-year-old client with type O blood B. A 42-year-old client with allergies C. A 58-year-old immune-suppressed client D. A 78-year-old client

A. A 34-year-old client with type O blood Hemolytic transfusion reactions are caused by blood type or Rh incompatibility. When blood that contains antigens different from the client's own antigens is infused, antigen-antibody complexes are formed in the client's blood. Type O is considered the universal donor, but not the universal recipient. *The client with allergies would be most susceptible to an allergic transfusion reaction. The immune-suppressed client would be most susceptible to a transfusion-associated graft-versus-host disease. The older adult client would be most susceptible to circulatory overload.*

An RN from pediatrics has "floated" to the medical-surgical unit. Which client is assigned to the float nurse? A. A 42-year-old with sickle cell disease receiving a transfusion of packed red blood cells B. A 50-year-old with pancytopenia needing assessment of risk factors for aplastic anemia C. A 55-year-old with folic acid deficiency anemia caused by alcohol abuse who needs counseling D. A 60-year-old with newly diagnosed polycythemia vera who needs teaching about the disease

A. A 42-year-old with sickle cell disease receiving a transfusion of packed red blood cells Because sickle cell disease is commonly diagnosed during childhood, the pediatric nurse will be familiar with the disease and with red blood cell transfusion; therefore, he or she should be assigned to the client with sickle cell disease. Aplastic anemia, folic acid deficiency, and polycythemia vera are problems more commonly seen in adult clients who should be cared for by nurses who are more experienced in caring for adults.

When preparing the client who is undergoing urography with contrast, the nurse plans to administer which medication before the procedure? c/o flank pain, dysuria, bilat knee pain with BUN/Cr: 54/2.4 a. Acetylcysteine b. Metformin c. Captopril d. Acetaminophen

A. Acetylcysteine (an antioxidant) may be used to prevent contrast-induced nephrotoxic effects; this client has kidney impairment demonstrated by increased creatinine.

The nurse is caring for a group of hospitalized clients. Which client is at greatest risk for infection and sepsis? A. An 18-year-old who had an emergency splenectomy B. A 22-year-old with recently diagnosed sickle cell anemia C. A 38-year-old with hemolytic anemia D. A 40-year-old alcoholic with liver disease

A. An 18-year-old who had an emergency splenectomy Removal of the spleen causes reduced immune function. Without a spleen, the client is less able to remove disease-causing organisms, and is at increased risk for infection. Sickle cell anemia causes pain and discomfort because of the changed cell morphology, so acute pain, especially at joints, is the greatest threat to this client. A low red blood cell count with hemolytic anemia can contribute to a client's risk for infection, but this client is more at risk for low oxygen levels and ensuing fatigue. The liver plays a role in blood coagulation, so this client is more at risk for coagulation problems than for infection.

A client has a bone marrow biopsy done. Which nursing intervention is the priority postprocedure? A. Applying pressure to the biopsy site B. Inspecting the site for ecchymoses C. Sending the biopsy specimens to the laboratory D. Teaching the client about avoiding vigorous activity

A. Applying pressure to the biopsy site The initial action should be to stop bleeding by applying pressure to the site. Inspecting for ecchymoses, sending specimens to the laboratory, and teaching the client about activity levels will be done after hemostasis has been achieved.

A client is exhibiting signs and symptoms of early shock. What is important for the nurse to do to support the psychosocial integrity of the client? (Select all that apply.) A. Ask family members to stay with the client. B. Call the health care provider. C. Increase IV and oxygen rates. D. Remain with the client. E. Reassure the client that everything is being done for him or her.

A. Ask family members to stay with the client. D. Remain with the client. E. Reassure the client that everything is being done for him or her. Having a familiar person nearby may provide comfort to the client. The nurse should remain with the client who is demonstrating physiologic deterioration. Offering genuine reassurance supports the client who is anxious. The health care provider should be notified, and increasing IV and oxygen rates may be needed, but these actions do not support the client's psychosocial integrity.

A 56-year-old client admitted with a diagnosis of acute myelogenous leukemia is prescribed IV cytosine arabinoside for 7 days and an infusion of daunorubicin for the first 3 days. What is the major side effect of this therapy? A. Bone marrow suppression B. Liver toxicity C. Nausea D. Stomatitis

A. Bone marrow suppression Intravenous cytosine arabinoside and daunorubicin are a commonly prescribed course of aggressive chemotherapy, and bone marrow suppression is a major side effect. The client is even more at risk for infection than before treatment began. Liver toxicity, nausea, and stomatitis are not the major problems with this therapy.

A client is admitted to the hospital with two of the systemic inflammatory response syndrome variables: temperature of 95° F (35° C) and high white blood cell count. Which intervention from the sepsis resuscitation bundle does the nurse initiate? A. Broad-spectrum antibiotics B. Blood transfusion C. Cooling baths D. NPO status

A. Broad-spectrum antibiotics Broad-spectrum antibiotics must be initiated within 1 hour of establishing diagnosis. A blood transfusion is indicated for low red blood cell count or low hemoglobin and hematocrit; transfusion is not part of the sepsis resuscitation bundle. Cooling baths are not indicated because the client is hypothermic, nor is this part of the sepsis resuscitation bundle. NPO status is not indicated for this client, nor is it part of the sepsis resuscitation bundle.

The RN working on an oncology unit has just received report on these clients. Which client should be assessed first? A. Client with chemotherapy-induced neutropenia who has just been admitted with an elevated temperature B. Client with lymphoma who will need administration of an antiemetic before receiving chemotherapy C. Client with metastatic breast cancer who is scheduled for external beam radiation in 1 hour D. Client with xerostomia associated with laryngeal cancer who needs oral care before breakfast

A. Client with chemotherapy-induced neutropenia who has just been admitted with an elevated temperature Neutropenia poses high risk for life-threatening sepsis and septic shock, which develop and progress rapidly in immune-suppressed people; the nurse should see the client with chemotherapy-induced neutropenia first. The client with lymphoma and the client with metastatic breast cancer are not in distress and can be assessed later. The client with dry mouth (xerostomia) can be assessed later, or the nurse can delegate mouth care to unlicensed assistive personnel.

The nursing assistant is concerned about a postoperative client with blood pressure (BP) of 90/60 mm Hg, heart rate of 80 beats/min, and respirations of 22 breaths/min. What does the supervising nurse do? A. Compare these vital signs with the last several readings. B. Request that the surgeon see the client. C. Increase the rate of intravenous fluids. D. Reassess vital signs using different equipment.

A. Compare these vital signs with the last several readings. Vital sign trends must be taken into consideration; a BP of 90/60 mm Hg may be normal for this client. Calling the surgeon is not necessary at this point, and increasing IV fluids is not indicated. The same equipment should be used when vital signs are taken postoperatively.

The nurse is teaching a client with vitamin B12 deficiency anemia about dietary intake. Which type of food does the nurse encourage the client to eat? A. Dairy products B. Grains C. Leafy vegetables D. Starchy vegetables

A. Dairy products Dairy products such as milk, cheese, and eggs will provide the vitamin B12 that the client needs. Grains, leafy vegetables, and starchy vegetables are not a source of vitamin B12.

Which factors indicate that a client's burn wounds are becoming infected? (Select all that apply.) A. Dry, crusty granulation tissue B. Elevated blood pressure C. Hypoglycemia D. Edema of the skin around the wound E. Tachycardia

A. Dry, crusty granulation tissue D. Edema of the skin around the wound E. Tachycardia Pale, boggy, dry, or crusted granulation tissue is a sign of infection, as is swelling or edema of the skin around the wound. Tachycardia is a systemic sign of infection. Hypotension, not elevated blood pressure, and hyperglycemia, not hypoglycemia, are systemic signs of infection.

A client with burn injuries states, "I feel so helpless." Which nursing intervention is most helpful for this client? A. Encouraging participation in wound care B. Encouraging visitors C. Reassuring the client that he or she will be fine D. Telling the client that these feelings are normal

A. Encouraging participation in wound care Encouraging participation in wound care will offer the client some sense of control. Encouraging visitors may be a good distraction, but will not help the client achieve a sense of control. Reassuring the client that he or she will be fine is neither helpful nor therapeutic. Telling the client that his or her feelings are normal may be reassuring, but does not address the client's issue of feeling helpless.

The client with which problem is at highest risk for hypovolemic shock? A. Esophageal varices B. Kidney failure C. Arthritis and daily acetaminophen use D. Kidney stone

A. Esophageal varices Esophageal varices are caused by portal hypertension; the portal vessels are under high pressure and are prone to rupture, causing massive upper gastrointestinal tract bleeding and hypovolemic shock. As the kidneys fail, fluid is typically retained, causing fluid volume excess, not hypovolemia. Nonsteroidal anti-inflammatory drugs such as naproxen and ibuprofen, not acetaminophen, predispose the client to gastrointestinal bleeding and hypovolemia. Although a kidney stone may cause hematuria, there is not generally massive blood loss or hypovolemia.

A client with partial-thickness burns of the face and chest caused by a campfire is admitted to the burn unit. The nurse plans to carry out which health care provider request first? A. Give oxygen per facemask. B. Infuse lactated Ringer's solution at 150 mL/hr. C. Give morphine sulfate 4 to 10 mg IV for pain control. D. Insert a 14 Fr retention catheter.

A. Give oxygen per facemask. Facial burns are frequently associated with upper airway inflammation. Administration of oxygen will assist in maintaining the client's tissue oxygenation at an optimal level. Although fluid hydration and pain control are important, the nurse's first priority is the client's airway. Monitoring output is important, but the nurse's first priority is the client's airway.

The nurse is reviewing the health history for an older adult client recently admitted to the burn unit with severe burns to the upper body from a house fire. The nurse plans to contact the health care provider if the client's history reveals which condition? A. Heart failure B. Diverticulitis C. Hypertension D. Emphysema

A. Heart failure A client's health history, including any pre-existing illnesses, must be known for appropriate management. Obtain specific information about the client's history of cardiac or kidney problems, chronic alcoholism, substance abuse, and diabetes mellitus. Any of these problems can influence fluid resuscitation. The stress of a burn injury can make a mild disease process worsen. In older clients, especially those with cardiac disease, a complicating factor in fluid resuscitation may be heart failure or myocardial infarction. Diverticulitis, hypertension and emphysema are important to be aware of in guiding treatment options. However, heart failure is the main concern when attempting to optimize this older client's fluid resuscitation.

The oncology nurse is caring for a group of clients receiving chemotherapy. The client with which sign/symptom is displaying bone marrow suppression? A. Hemoglobin of 7.4 and hematocrit of 21.8 B. Potassium level of 2.9 mEq/L and diarrhea C. 250,000 platelets/mm3 D. 5000 white blood cells/mm3

A. Hemoglobin of 7.4 and hematocrit of 21.8 Bone marrow suppression causes anemia, leukopenia, and thrombocytopenia; the client with a hemoglobin of 7.4 and hematocrit of 21.8 has anemia demonstrated by low hemoglobin and hematocrit. The client with diarrhea and a potassium level of 2.9 mEq/L has hypokalemia and electrolyte imbalance. The client with 250,000 platelets/mm3 and the client with 5000 white blood cells/mm3 demonstrate normal values.

To position a client's burned upper extremities appropriately, how does the nurse position the client's elbow? A. In a neutral position B. In a position of comfort C. Slightly flexed D. Slightly hyperextended

A. In a neutral position The neutral (extended) position is the correct placement of the elbow to prevent contracture development. Placing the elbow in a position of comfort is not the best placement because the client then usually wants to flex the joint, which increases the risk for contracture development. The slightly flexed position increases the risk for contracture development. The slightly hyperextended position is not indicated and can be painful.

Which urinary assessment information indicates the potential need for increased fluids in the client? A. Increased blood urea nitrogen B. Increased creatinine C. Pale-colored urine D. Decreased sodium

A. Increased blood urea nitrogen (BUN) can indicate dehydration.

The nurse explains to a client that which risk factor of those listed most likely contributed to the client's primary liver carcinoma? A. Infection with hepatitis B virus B. Consuming a diet high in animal fat C. Exposure to radon D. Familial polyposis

A. Infection with hepatitis B virus Hepatitis B and C are risk factors for primary liver cancer. Alcohol abuse is also a risk factor for the development of liver cancer. Consuming a diet high in animal fat may predispose a person to colon or breast cancer. Exposure to radon is a risk factor for lung cancer. Familial polyposis is a risk factor for colorectal cancer.

The client with which laboratory result is at risk for hemorrhagic shock? A. International normalized ratio (INR) 7.9 B. Partial thromboplastin time (PTT) 12.5 seconds C. Platelets 170,000/mm3 D. Hemoglobin 8.2 g/dL

A. International normalized ratio (INR) 7.9 Prolonged INR indicates that blood takes longer than normal to clot; this client is at risk for bleeding. PTT of 12.5 seconds and a platelet value of 170,000/mm3 are both normal and pose no risk for bleeding. Although a hemoglobin of 8.2 g/dL is low, the client could have severe iron deficiency or could have received medication affecting the bone marrow.

The nurse is caring for a client receiving chemotherapy who reports anorexia. Which measure does the nurse use to best monitor for cachexia? A. Monitor weight B. Trend red blood cells and hemoglobin and hematocrit C. Monitor platelets D. Observe for motor deficits

A. Monitor weight Cachexia results in extreme body wasting and malnutrition; severe weight loss is expected. Anemia and bleeding tendencies result from bone marrow suppression secondary to invasion of bone marrow by a cancer or a side effect of chemotherapy. Motor deficits result from spinal cord compression.

A client who is admitted with urolithiasis reports "spasms of intense flank pain, nausea, and severe dizziness." Which intervention does the nurse implement first? A. Administers morphine sulfate 4 mg IV B. Begins an infusion of metoclopramide (Reglan) 10 mg IV C. Obtains a urine specimen for urinalysis D. Starts an infusion of 0.9% normal saline at 100 mL/hr

A. Morphine administered IV will decrease the pain and the associated sympathetic nervous system reactions of nausea and hypotension.

When assessing the older adult, the nurse teaches the older adult that which age-related change causes nocturia? A. Decreased ability to concentrate urine B. Decreased production of antidiuretic hormone C. Increased production of erythropoietin D. Increased secretion of aldosterone

A. Nocturia may result from decreased kidney-concentrating ability associated with aging.

The nurse is assessing a newly admitted client with thrombocytopenia. Which factor needs immediate intervention? A. Nosebleed B. Reports of pain C. Decreased urine output D. Increased temperature

A. Nosebleed The client with thrombocytopenia has a high risk for bleeding. The nosebleed should be attended to immediately. The client's report of pain, decreased urine output, and increased temperature are not the highest priority.

A 32-year-old female with a urinary tract infection reports urinary frequency, urgency, and some discomfort upon urination. Her vital signs are stable except for a temperature of 100° F. She is started on antibiotics and sent home. She returns to the clinic 3 days later-with the same symptoms. When asked about the previous UTI and medication regimen, she states, "I only took the first dose because after that, I felt better." How does the nurse respond? A. "Not completing your medication can lead to return of your infection." B. "That means your treatment will be prolonged with this new infection." C. "This means you will now have to take two drugs instead of one." D. "What you did was okay; however, let's get you started on something else."

A. Not completing the drug regimen can lead to recurrence of an infection and to bacterial drug resistance.

The nurse reviews the medical record of a client with hemorrhagic shock, which contains the following information: Pulse 140 beats/min and thready, ABG respiratory acidosis, Blood pressure 60/40 mm Hg, Lactate level 7 mOsm/L, Respirations 40/min and shallow. All of these provider prescriptions are given for the client. Which does the nurse carry out first? A. Notify anesthesia for endotracheal intubation. B. Give Plasmanate 1 unit now. C. Give normal saline solution 250 mL/hr. D. Type and crossmatch for 4 units of packed red blood cells (PRBCs).

A. Notify anesthesia for endotracheal intubation. Establishing an airway is the priority in all emergency situations. Although administering Plasmanate and normal saline, and typing and crossmatching for 4 units of PRBCs are important actions, airway always takes priority.

A newly admitted client has deep partial-thickness burns. The nurse expects to see which clinical manifestations? A. Painful red and white wounds B. Painless, brownish yellow eschar C. Painful reddened blisters D. Painless black skin with eschar

A. Painful red and white wounds A painful red and white wound bed characterizes deep partial-thickness burns; blisters are rare. Painless, brownish yellow eschar characterizes a full-thickness burn. A painful reddened blister is seen with a superficial partial-thickness burn. Painless black skin with eschar is seen in a deep full-thickness burn.

The nurse is caring for a client with a burn injury who is receiving silver sulfadiazine (Silvadene) to the burn wounds. Which best describes the goal of topical antimicrobials? A. Reduction of bacterial growth in the wound and prevention of systemic sepsis B. Prevention of cross-contamination from other clients in the unit C. Enhanced cell growth D. Reduced need for a skin graft

A. Reduction of bacterial growth in the wound and prevention of systemic sepsis Topical antimicrobials such as silver sulfadiazine are an important intervention for infection prevention in burn wounds. Topical antimicrobials such as silver sulfadiazine do not prevent cross-contamination from other clients in the unit. They do not enhance cell growth, nor do they minimize the need the need for a skin graft.

Which statement made by the client who is receiving intravesicular instillations of BCG for bladder cancer indicates to the nurse that more teaching is needed? A. "Holding my urine for at least 8 hours after the treatment keeps the drug in contact with my bladder." B. "Drinking plenty of fluids during the evening after the treatment helps get the drug out of my system." C. "Sitting to urinate for 24 hours after treatment prevents exposure of other people to the drug." D. "Avoiding intercourse for 24 hours after treatment reduces my wife's exposure to the drug."

A. Retaining urine for that long distends the bladder and increases the risk that the BCG will be absorbed systemically. In addition, the discomfort may cause the client to urinate rapidly, leading to more splashing and potentially exposing others to the BCG.

A client recovering from an open reduction of the femur suddenly feels light-headed, with increased anxiety and agitation. Which key vital sign differentiates a pulmonary embolism from early sepsis? A. Temperature B. Pulse C. Respiration D. Blood pressure

A. Temperature A sign of early sepsis is low-grade fever. Both early sepsis and thrombus may cause tachycardia, tachypnea, and hypotension.

The nurse manager in a long-term care facility is developing a plan for primary and secondary prevention of colorectal cancer. Which tasks associated with the screening plan will be delegated to nursing assistants within the facility? A. Testing of stool specimens for occult blood B. Teaching about the importance of dietary fiber C. Referring clients for colonoscopy procedures D. Giving vitamin and mineral supplements

A. Testing of stool specimens for occult blood Testing of stool specimens for occult blood is done according to a standardized protocol and can be delegated to nursing assistants. Client education is within the scope of practice of the RN, not of the LPN or nursing assistant. Referral for further care is best performed by the RN. Administration of medications is beyond the nursing assistant's scope of practice and should be done by licensed nursing personnel.

The client has returned form a captopril renal scan. Which teaching should the nurse provide when the client returns? A."Arise slowly and call for assistance when ambulating." B. "I must measure your intake and output (I&O)." C. "We must save your urine because it is radioactive." D. "I must attach you to this cardiac monitor."

A. The drug can cause severe hypotension during and after the procedure. Warn him or her to avoid rapid position changes and of the risk for falling as a result of orthostatic (positional) hypotension.

Which assessments are most important for the nurse to perform when monitoring a client after a retrograde cystogram? A. Temperature and urine character B. Kidney tenderness and flank pain C. 24-hour urine volume, BUN and creatinine levels D. Angioedema and other indicators of systemic allergic response

A. The retrograde cystogram involves instilling a contrast dye directly into the bladder through the urethra. Infection is the primary concern from instruments entering the bladder. The dye does not enter the bloodstream, and systemic allergic responses do not occur. The kidneys are not involved in this imaging.

A nurse educates a group of women who have had frequent urinary tract infections (UTIs) about how to avoid recurrences of them. Which client statement shows correct understanding of what the nurse has taught? A. "I should be drinking at least 1.5 to 2.5 liters of fluids every day." B. "It is a good idea for me to reduce germs by taking a tub bath daily." C. "Trying to get to the bathroom to urinate every 6 hours is important for me." D. "Urinating 1000 mL on a daily basis is a good amount for me."

A. To reduce the number of UTIs, clients should be drinking a minimum of 1.5 to 2.5 liters of fluid (mostly water) each day.

When the nurse is counseling a 60-year-old African-American male client with all of these risk factors for lung cancer, teaching should focus most on which risk factor? A. Tobacco use B. Ethnicity C. Gender D. Increased age

A. Tobacco use Although all of these are risk factors for lung cancer, the client's tobacco use is the only factor that he can change. Ethnicity, gender, and increasing age are associated with lung cancer, but they are not modifiable risks.

Which clients are at immediate risk for hypovolemic shock? (Select all that apply.) A. Unrestrained client in motor vehicle accident B. Construction worker C. Athlete D. Surgical intensive care client E. 85-year-old with gastrointestinal virus

A. Unrestrained client in motor vehicle accident D. Surgical intensive care client E. 85-year-old with gastrointestinal virus The client who is unrestrained in a motor vehicle accident is prone to multiple trauma and bleeding. Surgical clients are at high risk for hypovolemic shock owing to fluid loss and hemorrhage. Older adult clients are prone to shock; a gastrointestinal virus results in fluid losses. Unless injured or working in excessive heat, the construction worker and the athlete are not at risk for hypovolemic shock; they may be at risk for dehydration.

The nurse is conducting a community health education class on diet and cancer risk reduction. What should be included in the discussion? (Select all that apply.) A. Limit sodium intake. B. Avoid beef and processed meats. C. Increase consumption of whole grains. D. Eat "colorful fruits and vegetables," including greens. E. Avoid gas-producing vegetables such as cabbage.

B, C, D Consuming bran and whole grains and avoiding red meat and processed foods such as lunchmeats can reduce cancer risk. Consuming foods high in vitamin A, including apricots, carrots, and leafy green and yellow vegetables, can also reduce cancer risk. Reducing sodium is helpful in the treatment of hypertension and heart and kidney failure; no evidence suggests that lowering of sodium intake decreases the incidence of cancer. Eating cruciferous vegetables such as broccoli, cauliflower, Brussels sprouts, and cabbage may actually reduce cancer risk.

What are the common cancers related to tobacco use? (Select all that apply.) A. Cardiac cancer B. Lung cancer C. Cancer of the tongue D. Skin cancer E. Cancer of the larynx

B, C, E Organs exposed to the carcinogens in tobacco (lungs, tongue, larynx) are the most likely to develop cancer. Oral cancer is also a risk with "smokeless" tobacco. The heart does not contain cells that divide; therefore, cardiac cancer is unlikely. Skin cancer generally is related to repeated sun and other ultraviolet exposure, such as that found with tanning beds.

When caring for a client who has had a colostomy created during treatment for colon cancer, which nursing actions help support the client in accepting changes in appearance or function? (Select all that apply.) A. Explain to the client that the colostomy is only temporary. B. Encourage the client to participate in changing the ostomy. C. Obtain a psychiatric consultation. D. Offer to have a person who is coping with a colostomy visit. E. Encourage the client and family members to express their feelings and concerns.

B, D, E Encouraging the client to participate in changing the ostomy is an appropriate way for the client to become familiar with the ostomy and its care. A visit from a person who is successfully coping with an ostomy can demonstrate to the client that many aspects of life can be the same after surgery. Offering to listen to feelings and concerns is part of a therapeutic relationship and therapeutic communication. Ostomies may be temporary for bowel rest, such as after a perforation, but are typically permanent for cancer treatment. Obtaining a psychiatric consultation may need to be done for clients with persistent depression, but would not be done immediately.

What are the typical clinical manifestations of anemia? (Select all that apply.) A. Decreased breath sounds B. Dyspnea on exertion C. Elevated temperature D. Fatigue E. Pallor F. Tachycardia

B, D, E, F Difficulty breathing—especially with activity—is common with anemia. Lower levels of hemoglobin carry less O2 to the cells of the body. Fatigue is a classic symptom of anemia; lowered O2 levels contribute to a faster pulse (i.e., cardiac rate) and tend to "wear out" a client's energy. Lowered O2 levels deliver less oxygen to all cells, making clients with anemia pale—especially their ears, nail beds, palms, and conjunctivae and around the mouth. Respiratory problems with anemia do not include changes in breath sounds; dyspnea and decreased oxygen saturation levels are present. Skin is cool to the touch, and an intolerance to cold is noted; elevated temperature would signify something additional, such as infection.

The nurse in the urology clinic is providing teaching for a female client with cystitis. Which of these should be included in the teaching plan? Select all that apply. A. Cleanse the perineum from back to front after using the bathroom. B. Try to take in 64 ounces of fluid each day. C. Be sure to complete the full course of antibiotics. D. If your urine remains cloudy, call the clinic. E. Expect some flank discomfort until the antibiotic has worked.

B,C,D: Between 64 and 100 ounces (2 to 3 liters) of fluid should be taken daily to dilute bacteria and prevent infection. Not completing the course of antibiotics could suppress the bacteria, but would not destroy all bacteria, causing the infection to resurface. For persistent symptoms of infection, the client should contact the provider. The perineal area should be cleansed from front to back or "clean to dirty" to prevent infection. Cystitis produces suprapubic symptoms; flank pain occurs with infection or inflammation of the kidney.

When assessing the client with pyelonephritis, which finding does the nurse anticipate will be present? Select all that apply. A. Suprapubic pain B. Vomiting C. Chills D. Dysuria E. Oliguria

B,C,D: Burning (dysuria), urgency, and frequency, Chills along with fever, Nausea and vomiting are symptoms of pyelonephritis. Suprapubic pain is indicative of cystitis, not kidney infection (pyelonephritis). Oliguria is related to kidney impairment from severe or long-standing pyelonephritis.

Pt is diagnosed with stress incontinence and is started on propantheline (Pro-Banthine). What interventions does the nurse suggest to alleviate the side effects of this anticholinergic drug? Select all that apply. A. Administer the drug at bedtime. B. Encourage increased fluids. C. Increase fiber. D. Limit the intake of dairy products. E. Offer hard candy for "dry" mouth.

B,C,E. Anticholinergics cause dry mouth and constipation. Increasing fluids and fiber will help.

Which interventions are helpful in preventing bladder cancer? Select all that apply. A. Drinking 2½ liters of fluid a day B. Showering after working with or around chemicals C. Stopping the use of tobacco D. Using pelvic floor muscle exercises E. Wearing a lead apron when working with chemicals F. Wearing gloves and a mask when working around chemicals and fumes

B,C,F: Certain chemicals are known to be carcinogenic in evaluating the risk for bladder cancer. Bathing after exposure and Protective gear is advisable.Tobacco use is one of the highest if not the highest risk factor in the development of bladder cancer.

The health care provider prescribes the drug, estrogen (Premarin). Which risks does the nurse tell the client to expect? Select all that apply. A. Dry mouth B. Endometrial cancer C. Increased intraocular pressure D. Thrombophlebitis E. Vaginitis

B,D: Estrogen use can increase the risk for endometrial cancer and the risk for thrombophlebitis. Women who smoke-especially-should not use this drug.

The nurse suspects metastasis from left breast cancer to the thoracic spine when the client has which symptom? A. Vomiting B. Back pain C. Frequent urination D. Cyanosis of the toes

B. Back pain Typical sites of breast cancer metastasis include bone (manifested by back pain), lung, liver, and brain. Signs of metastasis to the spine may include numbness, pain, paresthesias and tingling, and loss of bowel and bladder control, but not vomiting. Although frequent urination may be a sign of bladder cancer, incontinence is more indicative of spinal metastasis. Cyanosis of the toes indicates decreased tissue perfusion, often related to atherosclerotic disease.

A client with septic shock has been started on dopamine (Intropin) at 12 mcg/kg/min. Which response indicates a positive outcome? A. Hourly urine output 10 to 12 mL/hr B. Blood pressure 90/60 mm Hg and mean arterial pressure 70 mm Hg C. Blood glucose 245 mg/dL D. Serum creatinine 3.6 mg/dL

B. Blood pressure 90/60 mm Hg and mean arterial pressure 70 mm Hg Dopamine improves blood flow by increasing peripheral resistance, which increases blood pressure—a positive response in this case. Urine output less than 30 mL/hr or 0.5 mL/kg/hr and elevations in serum creatinine indicate poor tissue perfusion to the kidney and are a negative consequence of shock, not a positive response. Although a blood glucose of 245 mg/dL is an abnormal finding, dopamine increases blood pressure and myocardial contractility, not glucose levels.

A client with a burn injury due to a house fire is admitted to the burn unit. The client's family asks the nurse why the client received a tetanus toxoid injection on admission. What is the nurse's best response to the client's family member? A. "The last tetanus injection was less than 5 years ago." B. "Burn wound conditions promote the growth of Clostridium tetani." C. "The wood in the fire had many nails, which penetrated the skin." D. "The injection was prescribed to prevent infection from Pseudomonas."

B. "Burn wound conditions promote the growth of Clostridium tetani." Burn wound conditions promote the growth of Clostridium tetani, and all burn clients are at risk for this dangerous infection. Tetanus toxoid enhances acquired immunity to C. tetani, so this agent is routinely given when the client is admitted to the hospital. Regardless of when the last tetanus injection is given, it is still given on admission to prevent C. tetani. The fact that there were many nails in the wood in the fire is irrelevant. Tetanus toxoid injection does not prevent Pseudomonas infection.

The nurse is assessing the endurance level of a client in a long-term care facility. What question does the nurse ask to get this information? A. "Are your feet or hands cold, even when you are in bed?" B. "Do you feel more tired after you get up and go to the bathroom?" C. "How much exercise do you get?" D. "What is your endurance level?"

B. "Do you feel more tired after you get up and go to the bathroom?" Asking about feeling tired after using the bathroom is pertinent to the client's activity and provides a comparison. The specific activity helps the client relate to the question and provide needed answers. Asking about cold feet or hands does not address the client's endurance. The hospitalized client typically does not get much exercise; this would be a difficult assessment for a client in long-term care facility to make. Asking the client about his or her endurance level is too vague; the client may not know how to answer this question.

The nurse is assessing a client for hematologic function risks and seeks to determine whether there is a risk that cannot be reduced or eliminated. Which clinical health history question does the nurse ask to obtain this information? A. "Do you seem to have excessive bleeding or bruising?" B. "Does anyone in your family bleed a lot?" C. "Tell me what you eat in a day." D. "Where do you work?"

B. "Does anyone in your family bleed a lot?" An accurate family history is important because many disorders that affect blood and blood clotting are inherited; genetics cannot be changed. Excessive bleeding or bruising is a symptom, not a risk. Diet can affect risk, but it is a health behavior that can be changed. Work habits can be a risk, such as working near radiation, but these are behaviors that can be changed.

The nurse is educating a group of young women who have sickle cell disease (SCD). Which comment from a class member requires correction? A. "Frequent handwashing is an important habit for me to develop." B. "Getting an annual 'flu shot' would be dangerous for me." C. "I must take my penicillin pills as prescribed, all the time." D. "The pneumonia vaccine is protection that I need."

B. "Getting an annual 'flu shot' would be dangerous for me." The client with SCD should receive annual influenza and pneumonia vaccinations; this helps prevent the development of these infections, which could cause a sickle cell crisis. Handwashing is a very important habit for the client with SCD to develop because it reduces the risk for infection. Prophylactic penicillin is given to clients with SCD orally twice a day to prevent the development of infection.

A nurse is teaching a client with a neurogenic bladder to use intermittent self-catheterization for bladder emptying. Which client statement indicates a need for further clarification? A. "A small-lumen catheter will help prevent injury to my urethra." B. "I will use a new, sterile catheter each time I do the procedure." C. "My family members can be taught to help me if I need it." D. "Proper handwashing before I start the procedure is very important."

B. Catheters are cleaned and re-used. Proper handwashing and cleaning of the catheter have shown no increase in bacterial complications. Catheters are replaced when they show signs of deteriorating.

The nurse is caring for a client with burns to the face. Which statement by the client requires further evaluation by the nurse? A. "I am getting used to looking at myself." B. "I don't know what I will do when people stare at me." C. "I know that I will never look the way I used to, even after the scars heal." D. "My spouse does not stare at the scars as much now as in the beginning."

B. "I don't know what I will do when people stare at me." The statement about not knowing what to do when people stare indicates that the client is not coping effectively; the nurse should assist the client in exploring coping techniques. Visits from friends and short public appearances before discharge may help the client begin adjusting to this problem. The statement that the client is getting used to looking at himself or herself, the realization that he or she will always look different than before, and stating that the client's spouse doesn't stare at the scars as much all indicate that the client is coping effectively. Community reintegration programs can assist the psychosocial and physical recovery of the client with serious burns.

Which client statement indicates that stem cell transplantation that is scheduled to take place in his home is not a viable option? A. "I don't feel strong enough, but my wife said she would help." B. "I was a nurse, so I can take care of myself." C. "I will have lots of medicine to take." D. "We live 5 miles from the hospital."

B. "I was a nurse, so I can take care of myself." Stem cell transplantation in the home setting requires support, assistance, and coordination from others. The client cannot manage this type of care on his own. The client must be emotionally stable to be a candidate for this type of care. It is acceptable for the client's spouse to support the client undergoing this procedure. It is not unexpected for the client to be taking several prescriptions. Five miles is an acceptable distance from the hospital, in case of emergency.

The nurse reviews the chart of a client admitted with a diagnosis of glioblastoma with a T1NXM0 classification. Which explanation does the nurse offer when the client asks what the terminology means? A. "Two lymph nodes are involved in this tumor of the glial cells, and another tumor is present." B. "The brain tumor measures about 1 to 2 cm and shows no regional lymph nodes and no distant metastasis." C. "This type of tumor in the brain is small with some lymph node involvement; another tumor is present somewhere else in your body." D. "Glioma means this tumor is benign, so I will have to ask your health care provider the reason for the chemotherapy and radiation."

B. "The brain tumor measures about 1 to 2 cm and shows no regional lymph nodes and no distant metastasis." T1 means that the tumor is increasing in size to about 2 cm, and that no regional lymph nodes are present in the brain. M0 means that no distant metastasis has occurred. NX means that no regional lymph nodes can be assessed. A glioma is a benign tumor of the brain, but the client is diagnosed with a glioblastoma, which means a malignant tumor of the glial cells of the brain.

A client with anemia asks the nurse, "Do most people have the same number of red blood cells?" How does the nurse respond? A. "No, they don't." B. "The number varies with gender, age, and general health." C. "Yes, they do." D. "You have fewer red blood cells because you have anemia."

B. "The number varies with gender, age, and general health." Telling the client that the number of red blood cells (RBCs) varies with gender, age, and general health is the most educational and reasonable response to the client's question. Although telling the client that people do not have the same number of RBCs is true, it is not informative, and there is a better answer. While it may be true that the client has fewer red blood cells because of anemia, it does not answer the client's general question.

The nurse is reinforcing information about genetic counseling to a client with sickle cell disease who has a healthy spouse. What information does the nurse include? A. "Sickle cell disease will be inherited by your children." B. "The sickle cell trait will be inherited by your children." C. "Your children will have the disease, but your grandchildren will not." D. "Your children will not have the disease, but your grandchildren could."

B. "The sickle cell trait will be inherited by your children." The children of the client with sickle cell disease will inherit the sickle cell trait, but may not inherit the disease. If both parents have the sickle cell trait, their children could get the disease.

The nurse is teaching a client about what to expect during a bone marrow biopsy. Which statement by the nurse accurately describes the procedure? A. "The doctor will place a small needle in your back and will withdraw some fluid." B. "You may experience a crunching sound or a scraping sensation as the needle punctures your bone." C. "You will be alone because the procedure is sterile; we cannot allow additional people to contaminate the area." D. "You will be sedated, so you will not be aware of anything."

B. "You may experience a crunching sound or a scraping sensation as the needle punctures your bone." It is accurate to describe a crunching sound or scraping sensation. Proper expectations minimize the client's fear during the procedure. A very large-bore needle is used for a bone marrow biopsy, not a small needle; the puncture is made in the hip or in the sternum, not the back. The nurse, or sometimes a family member, is available to the client for support during a bone marrow biopsy. The procedure is sterile at the site of the biopsy, but others can be present without contamination at the site. A local anesthetic agent is injected into the skin around the site. The client may also receive a mild tranquilizer or a rapid-acting sedative (such as lorazepam [Ativan]) but will not be completely sedated.

Which client does the nurse assign as a roommate for the client with aplastic anemia? A. A 23-year-old with sickle cell disease who has two draining leg ulcers B. A 28-year-old with glucose-6-phosphate dehydrogenase (G6PD) deficiency anemia who is receiving mannitol (Osmitrol) C. A 30-year-old with leukemia who is receiving induction chemotherapy D. A 34-year-old with idiopathic thrombocytopenia who is taking steroids

B. A 28-year-old with glucose-6-phosphate dehydrogenase (G6PD) deficiency anemia who is receiving mannitol (Osmitrol) Because clients with aplastic anemia usually have low white blood cell counts that place them at high risk for infection, roommates such as the client with G6PD deficiency anemia should be free from infection or infection risk. The client with sickle cell disease has two draining leg ulcer infections that would threaten the diminished immune system of the client with aplastic anemia. The client with leukemia who is receiving induction chemotherapy and the client with idiopathic thrombocytopenia who is taking steroids are at risk for development of infection, which places the client with aplastic anemia at risk, too.

After reviewing the laboratory test results, the nurse calls the health care provider about which client? A. A 44-year-old receiving warfarin (Coumadin) with an international normalized ratio (INR) of 3.0 B. A 46-year-old with a fever and a white blood cell (WBC) count of 1500/mm3 C. A 49-year-old with hemophilia and a platelet count of 150,000/mm3 D. A 52-year-old who has had a hemorrhage with a reticulocyte count of 0.8%

B. A 46-year-old with a fever and a white blood cell (WBC) count of 1500/mm3 The client with a fever is neutropenic and is at risk for sepsis unless interventions such as medications to improve the WBC level and antibiotics are prescribed. The INR of 3.0 in the 44-year-old indicates a therapeutic warfarin level. A platelet count of 150,000/mm3 in the 49-year-old is normal. An elevated reticulocyte count in the 52-year-old is expected after hemorrhage.

Which clinical manifestation in the client with pyelonephritis indicates that treatment has been effective? A. Decreased urine output B. Decreased urine white blood cells C. Increased red blood cell count D. Increased urine specific gravity

B. A decreased presence of white blood cells indicates the eradication of infection.

The nurse is caring for a client who is receiving rituximab (Rituxan) for treatment of lymphoma. During the infusion, it is essential for the nurse to observe for which side effect? A. Alopecia B. Allergy C. Fever D. Chills

B. Allergy Allergy is the most common side effect of monoclonal antibody therapy (rituximab). Monoclonal antibody therapy does not cause alopecia. Although fever and chills are side effects of monoclonal antibody therapy, they would not take priority over an allergic response that could potentially involve the airway.

When caring for the client with hyperuricemia associated with tumor lysis syndrome (TLS), for which medication does the nurse anticipate an order? A. Recombinant erythropoietin (Procrit) B. Allopurinol (Zyloprim) C. Potassium chloride D. Radioactive iodine-131 (131I)

B. Allopurinol (Zyloprim) TLS results in hyperuricemia (elevation of uric acid in the blood), hyperkalemia, and other electrolyte imbalances; allopurinol decreases uric acid production and is indicated in TLS. Recombinant erythropoietin is used to increase red blood cell production and is not a treatment for hyperuricemia. Administering additional potassium is dangerous. Radioactive iodine-131 is indicated in the treatment of thyroid cancer, not TLS.

A nurse receives the change-of-shift report on four clients. Which client does the nurse decide to assess first? A. 26-year-old admitted 2 days ago with urosepsis with an oral temperature of 99.4° F (37.4° C) B. 28-year-old with urolithiasis who has been receiving morphine sulfate and has not voided for 8 hours C. 32-year-old admitted with hematuria and possible bladder cancer who is scheduled for cystoscopy D. 40-year-old with noninfectious urethritis who is reporting "burning" and has estrogen cream prescribed

B. Anuria may indicate urinary obstruction at the bladder neck or urethra and is an emergency because obstruction can cause acute kidney failure. The client may be oversedated and may not be aware of any discomfort caused by bladder distention.

Which problem in the clients below best demonstrates the highest risk for hypovolemic shock? A. Client receiving a blood transfusion B. Client with severe ascites C. Client with myocardial infarction D. Client with syndrome of inappropriate antidiuretic hormone (SIADH) secretion

B. Client with severe ascites Fluid shifts from vascular to intra-abdominal may cause decreased circulating blood volume and poor tissue perfusion. Volume depletion is only one reason why a person may require a blood transfusion; anemia is another. The client receiving a blood transfusion does not have as high a risk as the client with severe ascites. Myocardial infarction results in tissue necrosis in the heart muscle; no blood or fluid losses occur. Owing to excess antidiuretic hormone secretion, the client with SIADH will retain fluid and therefore is not at risk for hypovolemic shock.

What should the nurse teach the client who is undergoing a study using contrast media? A. "You will need to have anesthesia or sedation." B. "A feeling of heat or warmth occurs when the contrast is injected." C. "Expect your urine to have a pink or red tinge after the procedure." D. "You will not be able to eat or drink for 4 to 6 hours after the procedure."

B. Contrast medium causes a sensation of flushing, heat, or warmth as it circulates through the bloodstream.

Which laboratory result is seen in late sepsis? A. Decreased serum lactate B. Decreased segmented neutrophil count C. Increased numbers of monocytes D. Increased platelet count

B. Decreased segmented neutrophil count A decreased segmented neutrophil count is indicative of late sepsis. Serum lactate is increased in late sepsis. Monocytosis is usually seen in diseases such as tuberculosis and Rocky Mountain spotted fever. An increased platelet count does not indicate sepsis; late in sepsis, platelets may decrease due to consumptive coagulopathy.

The nurse is assessing a client with lung cancer. Which symptom does the nurse anticipate finding? A. Easy bruising B. Dyspnea C. Night sweats D. Chest wound

B. Dyspnea Dyspnea is a sign of lung cancer, as are cough, hoarseness, shortness of breath, bloody sputum, arm or chest pain, and dysphagia. Easy bruising is a nonspecific finding. Night sweats is a symptom of the lymphomas. A chest wound is not specific to lung cancer.

Which manifestation of an oncologic emergency requires the nurse to contact the health care provider immediately? A. New onset of fatigue B. Edema of arms and hands C. Dry cough D. Weight gain

B. Edema of arms and hands Edema of the arms and hands indicates worsening compression of the superior vena cava consistent with superior vena cava syndrome. The compression must be relieved immediately, often with radiation therapy, because death can result without timely intervention. New onset of fatigue may likely be an early manifestation of hypercalcemia, which usually develops slowly, but because it is an early manifestation, this is not the priority. Dry cough is not a manifestation that is specific to an oncologic emergency; however, it may be a side effect of chemotherapy. Weight gain could be an early sign of syndrome of inappropriate antidiuretic hormone; although this should be addressed, it is an early sign so it is not the priority.

Which intervention most effectively protects a client with thrombocytopenia? A. Avoiding the use of dentures B. Encouraging the use of an electric shaver C. Taking rectal temperatures D. Using warm compresses on trauma sites

B. Encouraging the use of an electric shaver The client with thrombocytopenia should be advised to use an electric shaver instead of a razor. Any small cuts or nicks can cause problems because of the prolonged clotting time. Dentures may be used by clients with thrombocytopenia as long as they fit properly and do not rub. To prevent rectal trauma, rectal thermometers should not be used. Oral or tympanic temperatures should be taken. Ice (not heat) should be applied to areas of trauma.

The nurse plans to administer an antibiotic to a client newly admitted with septic shock. What action does the nurse take first? A. Administer the antibiotic immediately. B. Ensure that blood cultures were drawn. C. Obtain signature for informed consent. D. Take the client's vital signs.

B. Ensure that blood cultures were drawn. Cultures must be taken to identify the organism for more targeted antibiotic treatment before antibiotics are administered. Antibiotics are not administered until after all cultures are taken. A signed consent is not needed for medication administration. Monitoring the client's vital signs is important, but the antibiotic must be administered within 1 to 3 hours; timing is essential.

The nurse on a burn unit has just received change-of-shift report about these clients. Which client does the nurse assess first? A. Adult client admitted a week ago with deep partial-thickness burns over 35% of the body who is reporting pain B. Firefighter with smoke inhalation and facial burns who has just arrived on the unit and whispers, "I can't catch my breath!" C. An electrician who suffered external burn injuries a month ago and is asking the nurse to contact the health care provider immediately about discharge plans D. Older adult client admitted yesterday with partial- and full-thickness burns over 40% of the body who is receiving IV fluids at 250 mL/hr

B. Firefighter with smoke inhalation and facial burns who has just arrived on the unit and whispers, "I can't catch my breath!" Smoke inhalation and facial burns are associated with airway inflammation and obstruction; the client with difficulty breathing needs immediate assessment and intervention. Although the client admitted a week ago with deep partial-thickness burns is reporting pain, this client does not require immediate assessment. The electrician who suffered burn injuries a month ago is stable and has been in the burn unit for a month, so the client's condition does not warrant that the nurse should assess this client first. The older adult client admitted yesterday with burns over 40% of the body is stable; he is receiving IV fluids and does not need to be assessed first.

When a diabetic client returns to the medical unit after IV urography, all of these interventions are prescribed. Which action will the nurse take first? A. Give lisper (Humalog) insulin, 12 units subcutaneously. B. Request a breakfast tray for the client. C. Infuse 0.45% normal saline at 125 mL/hr. D. Administer captopril (Capote).

C. Fluids are needed because the dye has an osmotic effect, causing dehydration and potential kidney failure.

The nurse is caring for a client with sickle cell disease. Which action is most effective in reducing the potential for sepsis in this client? A. Administering prophylactic drug therapy B. Frequent and thorough handwashing C. Monitoring laboratory values to look for abnormalities D. Taking vital signs every 4 hours, day and night

B. Frequent and thorough handwashing Prevention and early detection strategies are used to protect the client in sickle cell crisis from infection. Frequent and thorough handwashing is of the utmost importance. Drug therapy is a major defense against infections that develop in the client with sickle cell disease, but is not the most effective way that the nurse can reduce the potential for sepsis. Continually assessing the client for infection and monitoring the daily complete blood count with differential white blood cell count is early detection, not prevention. Taking vital signs every 4 hours will help with early detection of infection, but is not prevention.

A newly admitted client has an elevated reticulocyte count. Which disorder does the nurse suspect in this client? A. Aplastic anemia B. Hemolytic anemia C. Infectious process D. Leukemia

B. Hemolytic anemia An elevated reticulocyte count in an anemic client indicates that the bone marrow is responding appropriately to a decrease in the total red blood cell (RBC) mass and is prematurely destroying RBCs. Therefore, more immature RBCs are in circulation. Aplastic anemia is associated with a low reticulocyte count. A high white blood cell count is expected in clients with infection. A low white blood cell count is expected in clients with leukemia.

A 32-year-old client is recovering from a sickle cell crisis. His discomfort is controlled with pain medications and he is to be discharged. What medication does the nurse expect to be prescribed for him before his discharge? A. Heparin (Heparin) B. Hydroxyurea (Droxia) C. Tissue plasminogen activator (t-PA) D. Warfarin (Coumadin)

B. Hydroxyurea (Droxia) Hydroxyurea (Droxia) has been used successfully to reduce sickling of cells and pain episodes associated with sickle cell disease (SCD). Clients with SCD are not prescribed anticoagulants such as heparin or warfarin (Coumadin). t-PA is used as a "clot buster" in clients who have had ischemic strokes.

The nurse is transfusing 2 units of packed red blood cells to a postoperative client. What post-transfusion electrolyte imbalance does the nurse want to rule out? A. Hypercalcemia B. Hyperkalemia C. Hypomagnesemia D. Hyponatremia

B. Hyperkalemia During transfusion, some cells are damaged. These cells release potassium, thus raising the client's serum potassium level (hyperkalemia). This complication is especially common with packed cells and whole-blood products. High serum calcium levels, low magnesium levels, or low sodium levels are not expected with blood transfusions.

A client is in the resuscitation phase of burn injury. Which route does the nurse use to administer pain medication to the client? A. Intramuscular B. Intravenous C. Sublingual D. Topical

B. Intravenous During the resuscitation phase, the IV route is used for giving opioid drugs because of problems with absorption from the muscle and stomach. When these agents are given by the intramuscular or subcutaneous route, they remain in the tissue spaces and do not relieve pain. In addition, when edema is present, all doses are rapidly absorbed at once when the fluid shift is resolving. This delayed absorption can result in lethal blood levels of analgesics. The sublingual route may not be effective, and because the skin is too damaged, the topical route is not indicated for administering drugs to the client in the resuscitation phase of burn injury.

A recently admitted client who is in sickle cell crisis requests "something for pain." What does the nurse administer? A. Intramuscular (IM) morphine sulfate B. Intravenous (IV) hydromorphone (Dilaudid) C. Oral ibuprofen (Motrin) D. Oral morphine sulfate (MS-Contin)

B. Intravenous (IV) hydromorphone (Dilaudid) The client needs IV pain relief, and it should be administered on a routine schedule (i.e., before the client has to request it). Morphine is not administered intramuscularly (IM) to clients with sickle cell disease (SCD). In fact, all IM injections are avoided because absorption is impaired by poor perfusion and sclerosed skin. Nonsteroidal anti-inflammatory drugs may be used for clients with SCD for pain relief once their pain is under control; however, in a crisis, this choice of analgesic is not strong enough. Moderate pain may be treated with oral opioids, but this client is in a sickle cell crisis; IV analgesics should be used until his or her condition stabilizes.

How does the nurse caring for a client with septic shock recognize that severe tissue hypoxia is present? A. PaCO2 58 mm Hg B. Lactate 9.0 mmol/L C. Partial thromboplastin time 64 seconds D. Potassium 2.8 mEq/L

B. Lactate 9.0 mmol/L Poor tissue oxygenation at the cellular level causes anaerobic metabolism, with the by-product of lactic acid. Elevated partial pressure of carbon dioxide occurs with hypoventilation, which may be related to respiratory muscle fatigue, secretions, and causes other than hypoxia. Coagulation times reflect the ability of the blood to clot, not oxygenation at the cellular level. Elevation in potassium appears in septic shock due to acidosis; this value is decreased and is not consistent with septic shock.

Which clinical symptoms in a postoperative client indicate early sepsis with an excellent recovery rate if treated? A. Localized erythema and edema B. Low-grade fever and mild hypotension C. Low oxygen saturation rate and decreased cognition D. Reduced urinary output and increased respiratory rate

B. Low-grade fever and mild hypotension Low-grade fever and mild hypotension indicate very early sepsis, but with treatment, the probability of recovery is high. Localized erythema and edema indicate local infection. A low oxygen saturation rate and decreased cognition indicate active (not early) sepsis. Reduced urinary output and increased respiratory rate indicate severe sepsis.

How does the nurse recognize that a positive outcome has occurred when administering plasma protein fraction (Plasmanate)? A. Urine output 20 to 30 mL/hr for the last 4 hours B. Mean arterial pressure (MAP) 70 mm Hg C. Albumin 3.5 g/dL D. Hemoglobin 7.6 g/dL

B. Mean arterial pressure (MAP) 70 mm Hg Plasmanate expands the blood volume and helps maintain MAP greater than 65 mm Hg, which is a desired outcome in shock. Urine output should be 0.5 mL/kg/hr, or greater than 30 mL/hr. Albumin levels reflect nutritional status, which may be poor in shock states due to an increased need for calories. Plasmanate expands blood volume by exerting increasing colloid osmotic pressure in the bloodstream, pulling fluid into the vascular space; this does not improve an abnormal hemoglobin.

A postoperative client is admitted to the intensive care unit with hypovolemic shock. Which nursing action does the nurse delegate to an experienced nursing assistant? A. Obtain vital signs every 15 minutes. B. Measure hourly urine output. C. Check oxygen saturation. D. Assess level of alertness.

B. Measure hourly urine output. Monitoring hourly urine output is included in nursing assistant education and does not require special clinical judgment; the nurse evaluates the results. Obtaining vital signs, monitoring oxygen saturation, and assessing mental status in critically ill clients requires the clinical judgment of the critical care nurse because immediate intervention may be needed.

Which medication does the nurse plan to administer to a client before chemotherapy to decrease the incidence of nausea and vomiting? A. Morphine B. Ondansetron (Zofran) C. Naloxone (Narcan) D. Diazepam (Valium)

B. Ondansetron (Zofran) Ondansetron is a 5-HT3 receptor blocker that blocks serotonin to prevent nausea and vomiting. Morphine is a narcotic analgesic or opiate; it may cause nausea. Naloxone is a narcotic antagonist used for opiate overdose. Diazepam, a benzodiazepine, is an antianxiety medication only; lorazepam, another benzodiazepine, may be used for nausea.

Which wound assessment characteristics suggest a superficial partial-thickness burn injury? A. Black-brown coloration B. Painful C. Moderate to severe edema D. Absence of blisters

B. Painful Characteristics of a superficial partial-thickness burn injury include pink to red coloration, mild to moderate edema, pain, and blisters. A black-brown coloration is more suggestive of full-thickness burn injury. Moderate to severe edema and absence of blisters may be present with deep partial-thickness to full-thickness burn injuries.

A nursing student is caring for a client with open-wound burns. Which nursing interventions does the nursing student provide for this client? (Select all that apply.) A. Provides cushions and rugs for comfort B. Performs frequent handwashing C. Places plants in the client's room D. Performs gloved dressing changes E. Uses disposable dishes

B. Performs frequent handwashing D. Performs gloved dressing changes E. Uses disposable dishes Handwashing is the most effective technique for preventing infection. Gloves should be worn when changing dressings to reduce the risk for infection. Equipment is not shared with other clients to prevent the risk for infection. Disposable items (e.g., pillows, dishes) are used as much as possible. Cushions and rugs are difficult to clean and may harbor organisms, and so are not provided. To avoid exposure to Pseudomonas, having plants or flowers in the room is prohibited.

The nurse is reviewing complete blood count (CBC) data for a 76-year-old client. Which decreased value causes concern because it is not age-related? A. Hemoglobin level B. Platelet (thrombocyte) count C. Red blood cell (RBC) count D. White blood cell (WBC) response

B. Platelet (thrombocyte) count Platelet counts do not generally change with age. Hemoglobin levels in men and women fall after middle age; iron-deficient diets may play a role in this reduction. Total RBC and WBC counts (especially lymphocyte counts) are lower in older adults. The WBC count does not rise as high in response to infection in older adults as it does in younger people.

Which client problem does the nurse set as the priority for the client experiencing chemotherapy-induced peripheral neuropathy? A. Potential for lack of understanding related to side effects of chemotherapy B. Potential for injury related to sensory and motor deficits C. Potential for ineffective coping strategies related to loss of motor control D. Altered sexual function related to erectile dysfunction

B. Potential for injury related to sensory and motor deficits The highest priority is safety. Although knowing the side effects of chemotherapy may be helpful, the priority is the client's safety because of the lack of sensation or innervation to the extremities. The nurse should address the client's coping only after providing for safety. Erectile dysfunction may be a manifestation of peripheral neuropathy, but the priority is still the client's safety.

The nurse is caring for a client with end-stage ovarian cancer who needs clarification on the purpose of palliative surgery. Which outcome should the nurse teach the client is the goal of palliative surgery? A. Cure of the cancer B. Relief of symptoms or improved quality of life C. Allowing other therapies to be more effective D. Prolonging the client's survival time

B. Relief of symptoms or improved quality of life The focus of palliative surgery is to improve quality of life during the survival time. Curative surgery removes all cancer cells, visible and microscopic. Debulking is a procedure that removes some cancerous tissue, allowing other therapies to be more effective. Many therapies, such as surgery, chemotherapy, and biotherapy, increase the client's chance of cure and survival, but palliation improves quality of life.

The nurse assesses multiple clients who are receiving transfusions of blood components. Which assessment indicates the need for the nurse's immediate action? A. A partial thromboplastin time (PTT) that is 1.2 times normal in a client who received a transfusion of fresh-frozen plasma (FFP) B. Respiratory rate of 36 breaths/min in a client receiving red blood cells C. Sleepiness in a client who received diphenhydramine (Benadryl) as a premedication D. Temperature of 99.1° F (37.3° C) for a client with a platelet transfusion

B. Respiratory rate of 36 breaths/min in a client receiving red blood cells An increased respiratory rate indicates a possible hemolytic transfusion reaction; the nurse should quickly stop the transfusion and assess the client further. Because FFP is not usually given until the PTT is 1.5 times above normal, a PTT that is 1.2 times normal indicates that the FFP has had the desired response. Sleepiness is expected when Benadryl is administered. Temperature elevations are not an indication of an allergic reaction to a platelet transfusion, although the nurse may administer acetaminophen (Tylenol) to decrease the fever.

Which activity performed by the community health nurse best reflects primary prevention of cancer? A. Assisting women to obtain free mammograms B. Teaching a class on cancer prevention C. Encouraging long-term smokers to get a chest x-ray D. Encouraging sexually active women to get annual Papanicolaou (Pap) smears

B. Teaching a class on cancer prevention Primary prevention involves avoiding exposure to known causes of cancer; education assists clients with this strategy. Mammography is part of a secondary level of prevention, defined as screening for early detection. Chest x-ray is a method of detecting a cancer that is present—secondary prevention and early detection. A Pap smear is a means of detecting cervical cancer early—secondary prevention.

The RN is caring for a client who has just had a kidney biopsy. Which of these actions should the nurse perform first? A. Obtain BUN and creatinine. B. Position the client supine. C. Administer pain medications. D. Check urine for hematuria.

B. The client is positioned supine for several hours after a kidney biopsy to decrease the risk for hemorrhage.

A male client being treated for bladder cancer has a live virus compound instilled into his bladder as a treatment. What instructions does the nurse provide for postprocedure home care? A. "After 12 hours, your toilet should be cleaned with a 10% solution of bleach.". B. "Do not share your toilet with family members for the next 24 hours." C. "Please be sure to stand when you are urinating." D. "Your underwear worn during the procedure and for the first 12 hours afterward should be bagged and discarded."

B. The toilet should not be shared for 24 hours following the procedure because others using the toilet could be infected with the live virus that was instilled into the client. If the toilet must be shared, then specific cleaning precautions need to be taken each time the client uses the toilet. The best scenario is for the client not to share the toilet.

While assessing a client with overactive bladder, the nurse discovers the client also has the following health problems. Which health problem could be made worse by the drug tolterodine (Detrol)? A. Asthma B. Glaucoma C. Hypotension D. Diabetes mellitus

B. Tolterodine is an anticholinergic drug that can raise intraocular pressure and make some types of glaucoma worse. It is absolutely contraindicated for clients with uncontrolled narrow- or closed-angle glaucoma. It can be used with caution for clients who are being treated for open-angle glaucoma if the disease is well controlled.

A 67-year-old client who had an abdominal x-ray as part of pre-admission testing for a gastrointestinal problem has just been told that he has a horseshoe-shaped kidney. He is very upset, telling the nurse that he has never had any health problems until the past month and now feels that he is "falling apart." What is the nurse's best response? A. Remind him that it was lucky that he was being x-rayed anyway and that the problem was found at an early stage. B. Reassure him that it is unlikely that the kidney shape is important since he has not had other kidney problems. C. Ask him whether anyone else in his family has ever been diagnosed with a horseshoe-shaped kidney. D. Reassure him that his health care provider will request a consultation with a kidney specialist.

B. Variations in the number and shape of the kidneys are relatively common. Most variations are not harmful and do not require further assessment. The fact that this client is 67 years old and has had few health problems is a good indication that this is just an incidental finding on x-ray and has no meaning for his kidney health.

Which percussion technique does the nurse use to assess the client with reports of flank pain? A. Places fingers outstretched over the flank area and percusses with fingertips B. Places one hand with palm down flat over the flank area and uses the other fisted hand to thump the hand on the flank C. Places one hand with the palm up over the flank area and cups the other hand to percuss the hand on the flank D. Quickly taps the flank area with cupped hands

B. While the client assumes a sitting, side-lying, or supine position, form one of the hands into a clenched fist. Place the other hand flat over the costovertebral (CVA) angle of the client. Then, quickly deliver a firm thump to the hand over the CVA area.

Several clients have been brought to the emergency department after an office building fire. Which client is at greatest risk for inhalation injury? A. Middle-aged adult who is frantically explaining to the nurse what happened B. Young adult who suffered burn injuries in a closed space C. Adult with burns to the extremities D. Older adult with thick, tan-colored sputum

B. Young adult who suffered burn injuries in a closed space The client who suffered burn injuries in a closed space is at greatest risk for inhalation injury because the client breathed a greater concentration of confined smoke. Clients who experienced a fire typically have some type of respiratory distress. However, the client talking without difficulty demonstrates minimal respiratory distress. Extensive burns to the hands and face, not the extremities, would be a greater risk. Sputum would be carbonaceous, not tan, if the client had suffered inhalation injury.

Which of the following would alarm the nurse immediately after return of the client from the operating room for cystoscopy performed under conscious sedation? A. Pink-tinged urine B. Urinary frequency C. Temperature of 100.8 D. Client lethargic

C. Fever, chills, or an elevated white blood cell (WBC) count suggests infection after an invasive procedure; notify the provider.

A hematology unit is staffed by RNs, LPN/LVNs, and unlicensed assistive personnel (UAP). When the nurse manager is reviewing documentation of staff members, which entry indicates that the staff member needs education about his or her appropriate level of responsibility and client care? A. "Abdominal pain relieved by morphine 4 mg IV; client resting comfortably and denies problems. B.C., RN" B. "Ambulated in hallway for 40 feet and denies shortness of breath at rest or with ambulation. T.Y., LPN" C. "Client reporting increased shortness of breath; oxygen increased to 4 L by nasal cannula. M.N., UAP" D. "Vital signs 37.0° C, heart rate 60, respiratory rate 20, blood pressure 110/68, and oximetry 98% on room air. L.D., UAP"

C. "Client reporting increased shortness of breath; oxygen increased to 4 L by nasal cannula. M.N., UAP" Determination of the need for oxygen and administration of oxygen should be done by licensed nurses who have the education and scope of practice required to administer it. All other documentation entries reflect appropriate delegation and assignment of care.

A 52-year-old client relates to the nurse that she has never had a mammogram because she is terrified that she will have cancer. Which response by the nurse is therapeutic? A. "Don't worry, most lumps are discovered by women during breast self-examination." B. "Does anyone in your family have breast cancer?" C. "Finding a cancer in the early stages increases the chance for cure." D. "Have you noticed a lump or thickening in your breast?"

C. "Finding a cancer in the early stages increases the chance for cure." Providing truthful information addresses the client's concerns. Mammography can detect lumps smaller than those discovered by palpation. Asking about family history or symptoms is not therapeutic because it does not address the client's fear of cancer.

The nurse is caring for a client with burns. Which question does the nurse ask the client and family to assess their coping strategies? A. "Do you support each other?" B. "How do you plan to manage this situation?" C. "How have you handled similar situations before?" D. "Would you like to see a counselor?"

C. "How have you handled similar situations before?" Asking how the client and family have handled similar situations in the past assesses whether the client's and the family's coping strategies may be effective. "Yes-or-no" questions such as "Do you support each other?" are not very effective in extrapolating helpful information. The client and family in this situation probably are overwhelmed and may not know how they will manage; asking them how they plan to manage the situation does not assess coping strategies. Asking the client and the family if they would like to see a counselor also does not assess their coping strategies.

The nurse is assessing an adult client's endurance in performing activities of daily living (ADLs). What question does the nurse ask the client? A. "Can you prepare your own meals?" B. "Has your weight changed by 5 pounds or more this year?" C. "How is your energy level compared with last year?" D. "What medications do you take daily, weekly, and monthly?"

C. "How is your energy level compared with last year?" Asking the client how his or her energy level compares with last year is an activity exercise question that correctly assesses endurance compared with self-assessment in the past. It is most likely to provide data about the client's ability and endurance for ADLs. The client may never have been able to prepare his or her own meals, and the ability to prepare meals does not really address endurance. The question about weight change addresses nutrition and metabolic needs, rather than ADL performance. The question about how often the client takes medication addresses nutrition and metabolic needs and focuses on health maintenance through the use of drugs, not on the client's ability to perform ADLs.

Which statement made by a client allows the nurse to recognize whether the client receiving brachytherapy for ovarian cancer understands the treatment plan? A. "I may lose my hair during this treatment." B. "I must be positioned in the same way during each treatment." C. "I will have a radioactive device in my body for a short time." D. "I will be placed in a semiprivate room for company."

C. "I will have a radioactive device in my body for a short time." Brachytherapy refers to short-term insertion of a radiation source. Side effects of radiation therapy are site-specific; this client is unlikely to experience hair loss from treating ovarian cancer with radiation. The client undergoing teletherapy (external beam radiation), not brachytherapy, must be positioned precisely in the same position each time. The client who is receiving brachytherapy must be in a private room.

A client on anticoagulant therapy is being discharged. Which statement indicates that the client has a correct understanding of this therapy's purpose or action? A. "It is to dissolve blood clots." B. "It might cause me to get injured more often." C. "It should prevent my blood from clotting." D. "It will thin my blood."

C. "It should prevent my blood from clotting." Anticoagulants work by interfering with one or more steps involved in the blood clotting cascade. Thus, these agents prevent new clots from forming and limit or prevent extension of formed clots. Anticoagulants do not dissolve clots, fibrinolytics do. Anticoagulants do not cause more injuries, but may cause more bleeding and bruising when the client is injured. Anticoagulants do not cause any change in the thickness or viscosity of the blood.

A client who is scheduled to undergo radiation for prostate cancer is admitted to the hospital by the nurse. Which statement by the client is most important to communicate to the health care provider? A. "I am allergic to iodine." B. "My urinary stream is very weak." C. "My legs are numb and weak." D. "I am incontinent when I cough."

C. "My legs are numb and weak." Numbness and weakness should be reported to the physician because paralysis caused by spinal cord compression can occur. Prostate cancer may frequently metastasize to the bone, specifically the spine. Allergy to iodine should be reported when contrast media will be used, but dye is not used in radiation therapy. A weak urinary stream and incontinence are common clinical manifestations of prostate cancer. Incontinence associated with coughing is typical of stress incontinence and is not a complication of cancer.

The clinic nurse is discharging a 20-year-old client who had a bone marrow aspiration performed. What does the nurse advise the client to do? A. "Avoid contact sports or activity that may traumatize the site for 24 hours." B. "Inspect the site for bleeding every 4 to 6 hours." C. "Place an ice pack over the site to reduce the bruising." D. "Take a mild analgesic, such as two aspirin, for pain or discomfort at the site."

C. "Place an ice pack over the site to reduce the bruising." Ice to the site will help limit bruising and tissue damage during the first 24 hours after the procedure. Contact sports and traumatic activity must be excluded for 48 hours, or 2 days. The client should carefully monitor the site every 2 hours for the first 24 hours after the procedure. A mild analgesic is appropriate, but it should be aspirin-free; acetaminophen (Tylenol) would be a good choice.

A client who was the sole survivor of a house fire says, "I feel so guilty. Why did I survive?" What is the best response by the nurse? A. "Do you want to pray about it?" B. "I know, and you will have to learn to adapt to a new body image." C. "Tell me more." D. "There must be a reason."

C. "Tell me more." Asking the client to tell the nurse more encourages therapeutic grieving. Offering to pray with the client assumes that prayer is important to the client and does not allow for grieving; the nurse should never assume that the client is religious. The response, "I know, and you will have to learn to adapt to a new body image" only serves to add stress to the client's situation. The response, "There must be a reason" minimizes the grieving process by not allowing the client to express his or her concerns.

A client with a low platelet count asks why platelets are important. How does the nurse answer? A. "Platelets make your blood clot." B. "Blood clotting is prevented by your platelets." C. "The clotting process begins with your platelets." D. "Your platelets finish the clotting process."

C. "The clotting process begins with your platelets." Platelets begin the blood clotting process by forming platelet plugs, but these platelet plugs are not clots and cannot provide complete hemostasis. Platelets do not clot blood; they are a part of the clotting process or cascade of coagulation. Platelets do not prevent the blood from clotting; rather they function to help blood form clots. Platelets do not finish the clotting process, they begin it.

A client with leukemia is being discharged from the hospital. After hearing the nurse's instructions to keep regularly scheduled follow-up provider appointments, the client says, "I don't have transportation." How does the nurse respond? A. "A pharmaceutical company might be able to help." B. "I might be able to take you." C. "The local American Cancer Society may be able to help." D. "You can take the bus."

C. "The local American Cancer Society may be able to help." Many local units of the American Cancer Society offer free transportation to clients with cancer, including those with leukemia. Suggesting a pharmaceutical company is not the best answer; drug companies typically do not provide this type of service. Although the nurse offering to take the client is compassionate, it is not appropriate for the nurse to offer the client transportation. Telling the client to take the bus is dismissive and does not take into consideration the client's situation (e.g., the client may live nowhere near a bus route).

A newly graduated RN has just finished a 6-week orientation to the oncology unit. Which client is most appropriate to assign to the new graduate? A. A 30-year-old with acute lymphocytic leukemia who will receive combination chemotherapy today B. A 40-year-old with chemotherapy-induced nausea and vomiting who has had no urine output for 16 hours C. A 45-year-old with pancytopenia who will require IV administration of erythropoietin (Procrit) D. A 72-year-old with tumor lysis syndrome who is receiving normal saline IV at a rate of 250 mL/hr

C. A 45-year-old with pancytopenia who will require IV administration of erythropoietin (Procrit) A new nurse after a 6-week oncology orientation possesses the skills to care for clients with pancytopenia and with administration of medications to correct anemia. The clients with acute lymphocytic leukemia and chemotherapy-induced nausea are complex clients requiring a nurse certified in chemotherapy administration. The client with tumor lysis syndrome has complicated needs for assessment and care and should be cared for by an RN with more oncology experience.

Which client does the medical unit charge nurse assign to an LPN/LVN? A. A 23-year-old scheduled for a bone marrow biopsy with conscious sedation B. A 35-year-old with a history of a splenectomy and a temperature of 100.9° F (38.3° C) C. A 48-year-old with chronic microcytic anemia associated with alcohol use D. A 62-year-old with atrial fibrillation and an international normalized ratio of 6.6

C. A 48-year-old with chronic microcytic anemia associated with alcohol use Chronic microcytic anemia is not considered life-threatening and can be assigned to an LPN/LVN. The clients with a bone marrow biopsy with conscious sedation, a history of splenectomy and a temperature, and atrial fibrillation require more complex assessment or nursing care and should be assigned to RN staff members.

Which instruction does the nurse give the client who needs a clean catch urine specimen? A. Save all urine for 24 hours. B. I will collect the first specimen of the morning. C. Do not touch the inside of the container. D. You will receive an isotope injection, then I will collect your urine.

C. A clean catch specimen is used to obtain urine for culture and sensitivity of organisms present; contamination by the client's hands will alter the specimen and results.

Which statement made by a client who has acute pyelonephritis indicates to the nurse correct understanding of the antibiotic therapy? A. "If my temperature is normal for 3 days in a row, the infection is gone and I can stop taking the drug." B. "If my temperature goes above 100° for 2 days, I should take double the dose of the drug." C. "Even if I feel completely well, I should take the drug exactly as prescribed until it is gone." D. "I should notify my prescriber to change the medication if I develop diarrhea while taking this drug."

C. Antibiotic therapy is most effective when taken for the entire course and not just when symptoms are present. Most antibiotic therapy results in some degree of diarrhea. Although additional drugs may be needed to control this side effect, it is usually unnecessary to stop the drug.

When reviewing the medical record for a client with polycystic kidney disease who is scheduled for computed tomography (CT) angiography with contrast, it is essential for the nurse to perform which intervention? A. Obtain a thyroid-stimulating hormone (TSH) level. B. Report the blood urea nitrogen (BUN) and creatinine. C. Hold the metformin 24 hours before and on the day of the procedure. D. Notify provider regarding blood glucose and hemoglobin A1c values.

C. Before studies with contrast media are performed, the nurse must withhold metformin, which may cause lactic acidosis.

A 53-year-old postmenopausal woman is diagnosed with stress incontinence. What does the nurse tell the client about how certain drugs may be able to help with her? A. "They can relieve your anxiety associated with incontinence." B. "They help your bladder to empty." C. "They may be used to improve urethral resistance." D. "They decrease your bladder's tone."

C. Bladder pressure is greater than urethral resistance; drugs may be used to improve urethral resistance.

The client prescribed cephalexin (Keflex) for cystitis reports that she has had a severe allergic reaction to penicillin in the past. What is the nurse's best action? A. Reassure the client that Keflex is not penicillin. B. Place an allergy alert band on the client's wrist. C. Notify the prescriber before administering the first Keflex dose. D. Highlight this important information in the client's medical record.

C. Cephalexin is a cephalosporin and has a chemical structure very similar to the structure of penicillin. Often a person who is allergic to penicillin is also allergic to cephalosporins. Even if the prescriber wishes to proceed with cephalosporin therapy, he or she may first prescribe premedication to reduce the risk for an allergic response

Which finding alarms the nurse when caring for a client receiving chemotherapy who has a platelet count of 17,000/mm3? A. Increasing shortness of breath B. Diminished bilateral breath sounds C. Change in mental status D. Weight gain of 4 pounds in 1 day

C. Change in mental status A change in mental status could result from spontaneous bleeding; in this case, a cerebral hemorrhage may have developed. Increasing shortness of breath is typically related to anemia, not to thrombocytopenia. Diminished breath sounds may be related to many factors, including poor respiratory excursion, infection, and atelectasis, which is not related to thrombocytopenia. A large weight gain in a short period may be related to kidney or heart failure; bleeding is the major complication of thrombocytopenia.

A client with burn injuries is being admitted. Which priority does the nurse anticipate within the first 24 hours? A. Range-of-motion exercises B. Emotional support C. Fluid resuscitation D. Sterile dressing changes

C. Fluid resuscitation The client will require fluid resuscitation because fluid does not stay in the vessels after a burn injury. Range-of-motion exercise is not the priority for this client. Although emotional support and sterile dressing changes are important, they are not the priority during the resuscitation phase of burn injury.

A 56-year-old client admitted with a diagnosis of acute myelogenous leukemia (AML) is prescribed IV cytosine arabinoside for 7 days and an infusion of daunorubicin for the first 3 days. An infection develops. What knowledge does the nurse use to determine that the appropriate antibiotic has been prescribed for this client? A. Evaluating the client's liver function tests (LFTs) and serum creatinine levels B. Evaluating the client's white blood cell (WBC) count level C. Checking the culture and sensitivity test results to be certain that the requested antibiotic is effective against the organism causing the infection D. Recognizing that vancomycin (Vancocin) is the drug of choice used to treat all infections in clients with AML

C. Checking the culture and sensitivity test results to be certain that the requested antibiotic is effective against the organism causing the infection Checking the culture and sensitivity test results to be certain that the requested antibiotic is effective against the organism causing the infection is the best action to take. Drug therapy is the main defense against infections that develop in clients undergoing therapy for AML. Agents used depend on the client's sensitivity to various antibiotics for the organism causing the infection. Although LFTs and kidney function tests may be influenced by antibiotics, these tests do not determine the effectiveness of the antibiotic. Although the WBC count is elevated in infection, this test does not influence which antibiotic will be effective in fighting the infection. Vancomycin may not be effective in all infections; culturing of the infection site and determining the organism's sensitivity to a cohort of drugs are needed, which will provide data on drugs that are capable of eradicating the infection in this client.

Which instruction is most appropriate for the nurse to convey to the client with chemotherapy-induced neuropathy? A. Bathe in cold water. B. Wear cotton gloves when cooking. C. Consume a diet high in fiber. D. Make sure shoes are snug.

C. Consume a diet high in fiber. A high-fiber diet will assist with constipation due to neuropathy. The client should bathe in warm water, not hotter than 96° F. Cotton gloves may prevent harm from scratching; protective gloves should be worn for washing dishes and gardening. Wearing cotton gloves while cooking can increase the risk for burns. Shoes should allow sufficient length and width to prevent blisters. Shoes that are snug can increase the risk for blisters in a client with peripheral neuropathy.

Which action does the nurse delegate to unlicensed assistive personnel (UAP) who are assisting with the care of a female client with anemia? A. Asking the client about the amount of blood loss with each menstrual period B. Checking for sternal tenderness while applying fingertip pressure C. Determining the respiratory rate before and after the client walks 20 feet D. Monitoring her oral mucosa for pallor, bleeding, or ulceration

C. Determining the respiratory rate before and after the client walks 20 feet Assessment of the respiratory rate before and after ambulation is within the scope of practice for UAP; UAP will report this information to the RN. Asking the client about the amount of blood loss with each menstrual period, checking for sternal tenderness, and monitoring oral mucosa require skilled assessment techniques and knowledge of normal parameters and should be done by the RN.

The nurse is caring for a client in the refractory stage of cardiogenic shock. Which intervention does the nurse consider? A. Admission to rehabilitation hospital for ambulatory retraining B. Collaboration with home care agency for return to home C. Discussion with family and provider regarding palliative care D. Enrollment in a cardiac transplantation program

C. Discussion with family and provider regarding palliative care In this irreversible phase, therapy is not effective in saving the client's life, even if the cause of shock is corrected and mean arterial pressure temporarily returns to normal. A discussion on palliative care should be considered. Rehabilitation or returning home is unlikely. The client with sustained tissue hypoxia is not a candidate for organ transplantation.

Which strategy does the nurse include when teaching a college student about fire prevention in the dormitory room? A. Use space heaters to reduce electrical costs. B. Check water temperature before bathing. C. Do not smoke in bed. D. Wear sunscreen.

C. Do not smoke in bed. Smoking in bed increases the risk for fire because the person could fall asleep. Use of space heaters may increase the risk for fire, especially if they are knocked over and left unattended. Checking water temperature does not prevent fires, but it should be checked if the client has reduced sensation in the hands or feet. Sunscreen is advised to prevent sunburn.

A nurse is instructing an older adult female client about interventions to decrease the risk for cystitis. Which client comment indicates that the teaching was effective? A. "I must avoid drinking carbonated beverages." B. "I need to douche vaginally once a week." C. "I should drink 2½ liters of fluid every day." D. "I will not drink fluids after 8 PM each evening."

C. Drinking 2½ liters of fluid a day flushes out the urinary system and helps reduce the risk for cystitis.

A 32-year-old female with a urinary tract infection reports urinary frequency, urgency, and some discomfort upon urination. Her vital signs are stable except for a temperature of 100° F. Which drug does the nurse expect the health care provider to prescribe? A. Nitrofurantoin after intercourse B. Premarin C. Trimethoprim/sulfamethoxazole D. Trimethoprim with intercourse

C. Guidelines indicate that a 3-day course of trimethoprim/sulfamethoxazole or fosfomycin is effective in treating uncomplicated, community-acquired UTI in women.

A cognitively impaired client has urge incontinence. Which method for achieving continence does a nurse include in the client's care plan? A. Bladder training B. Credé method C. Habit training D. Kegel exercises

C. Habit training (scheduled toileting) will be most effective in reducing incontinence for a cognitively impaired client because the caregiver is responsible for helping the client to a toilet on a scheduled basis.

When caring for a client receiving chemotherapy, the nurse plans care during the nadir of bone marrow activity to prevent which complication? A. Drug toxicity B. Polycythemia C. Infection D. Dose-limiting side effects

C. Infection The lowest point of bone marrow function is referred to as the nadir; risk for infection is highest during this phase. Drug toxicity can develop when drug levels exceed peak concentrations. Polycythemia refers to an increase in the number of red blood cells; typically chemotherapy causes reduction of red blood cells or anemia. Dose limiting side effects occur when the dose or frequency of chemotherapy need to be altered or held, such as in the case of severe neutropenia or neurologic dysfunction .

The nurse is infusing platelets to a client who is scheduled for a hematopoietic stem cell transplant. What procedure does the nurse follow? A. Administer intravenous corticosteroids before starting the transfusion. B. Allow the platelets to stabilize at the client's bedside for 30 minutes. C. Infuse the transfusion over a 15- to 30-minute period. D. Set up the infusion with the standard transfusion Y tubing.

C. Infuse the transfusion over a 15- to 30-minute period. The volume of platelets—200 or 300 mL (standard amount)—needs to be infused rapidly over a 15- to 30-minute period. Administering steroids is not standard practice in administering platelets. Platelets must be administered immediately after they are received; they are considered to be quite fragile. A special transfusion set with a smaller filter and shorter tubing is used to get the platelets into the client quickly and efficiently.

Which nursing activity illustrates proper aseptic technique during catheter care? A. Applying Betadine ointment to the perineal area after catheterization B. Irrigating the catheter daily C. Positioning the collection bag below the height of the bladder D. Sending a urine specimen to the laboratory for testing

C. Keep urine collection bags below the level of the bladder at all times. Elevating the collection bag above the bladder causes reflux of pathogens from the bag into the urinary tract.

When caring for the client with uremia, the nurse assesses for which of these symptoms? A. Tenderness at the costovertebral angle (CVA) B. Cyanosis of the skin C. Nausea and vomiting D. Insomnia

C. Manifestations of uremia include anorexia, nausea, vomiting, weakness, and fatigue.

The nurse is caring for a client with neutropenia who has a suspected infection. Which intervention does the nurse implement first? A. Hydrate the client with 1000 mL of IV normal saline. B. Initiate the administration of prescribed antibiotics. C. Obtain requested cultures. D. Place the client on Bleeding Precautions.

C. Obtain requested cultures. Obtaining cultures to identify the infectious agent correctly is the priority for this client. Hydrating the client is not the priority. Administering antibiotics is important, but antibiotics should always be started after cultures are obtained. Placing the client on Bleeding Precautions is unnecessary.

Which type of incontinence benefits from pelvic floor muscle (Kegel) exercise? A. Functional B. Overflow C. Stress D. Urge

C. Pelvic floor (Kegel) exercise therapy for women with stress incontinence strengthens the muscles of the pelvic floor, thereby helping decrease the occurrence of incontinence

A 32-year-old client recovering from a sickle cell crisis is to be discharged. The nurse says, "You and all clients with sickle cell disease are at risk for infection because of your decreased spleen function. For this reason, you will most likely be prescribed an antibiotic before discharge." Which drug does the nurse anticipate the health care provider will request? A. Cefaclor (Ceclor) B. Gentamicin (Garamycin) C. Penicillin V (Pen-V K) D. Vancomycin (Vancocin)

C. Penicillin V (Pen-V K) Prophylactic therapy with twice-daily oral penicillin reduces the incidence of pneumonia and other streptococcal infections and is the correct drug to use. It is a standard protocol for long-term prophylactic use in clients with sickle cell disease. Cefaclor (Ceclor) and vancomycin (Vancocin) are antibiotics more specific for short-term use and would be inappropriate for this client. Gentamicin (Garamycin) is a drug that can cause liver and kidney damage with long-term use.

Which problem places a client at highest risk for sepsis? A. Pernicious anemia B. Pericarditis C. Post kidney transplant D. Client owns an iguana

C. Post kidney transplant The post-kidney transplant client will need to take lifelong immune suppressant therapy and is at risk for infection from internal and external organisms. Pernicious anemia is related to lack of vitamin B12, not to bone marrow failure (aplastic anemia), which would place the client at risk for infection. Inflammation of the pericardial sac is an inflammatory condition that does not pose a risk for septic shock. Although owning pets, especially cats and reptiles, poses a risk for infection, the immune-suppressed kidney transplant client has a very high risk for infection, sepsis, and death.

The client is in the emergency department (ED) for an inability to void and for bladder distention. What is most important for the nurse to provide to the client? A. Increased oral fluids B. IV fluids C. Privacy D. Health history forms

C. Provide privacy, assistance, and voiding stimulants, such as warm water over the perineum, as needed, for the client with urinary problems.

When caring for the client with chemotherapy-induced mucositis, which intervention will be most helpful? A. Administering a biological response modifier B. Encouraging oral care with commercial mouthwash C. Providing oral care with a disposable mouth swab D. Maintaining NPO until the lesions have resolved

C. Providing oral care with a disposable mouth swab The client with mucositis would benefit most from oral care; mouth swabs are soft and disposable and therefore clean and appropriate to provide oral care. Biological response modifiers are used to stimulate bone marrow production of immune system cells; mucositis or sores in the mouth will not respond to these medications. Commercial mouthwashes should be avoided because they may contain alcohol or other drying agents that may further irritate the mucosa. Keeping the client NPO is not necessary because nutrition is important during cancer treatment; a local anesthetic may be prescribed for comfort.

In assessing a client in the rehabilitative phase of burn therapy, which priority problem does the nurse anticipate? A. Intense pain B. Potential for inadequate oxygenation C. Reduced self-image D. Potential for infection

C. Reduced self-image In the rehabilitative phase of burn therapy, the client is discharged and his or her life is not the same. A priority problem of reduced self-image is expected. Intense pain and potential for inadequate oxygenation are relevant in the resuscitation phase of burn injury. Potential for infection is relevant in the acute phase of burn injury.

The nurse is to administer packed red blood cells to a client. How does the nurse ensure proper client identification? A. Asks the client's name B. Checks the client's armband C. Reviews all information with another registered nurse D. Verifies the client's room number

C. Reviews all information with another registered nurse With another registered nurse, verify the client by name and number, check blood compatibility, and note expiration time. Human error is the most common cause of ABO incompatibility reactions, even for experienced nurses. Asking the client's name and checking the client's armband are not adequate for identifying the client before transfusion therapy. Using the room number to verify client identification is never appropriate.

Which clinical manifestation is indicative of wound healing for a client in the acute phase of burn injury? A. Pale, boggy, dry, or crusted granulation tissue B. Increasing wound drainage C. Scar tissue formation D. Sloughing of grafts

C. Scar tissue formation Indicators of wound healing include the presence of granulation, re-epithelialization, and scar tissue formation. Pale, boggy, dry, or crusted granulation tissue is indicative of infection, as are increasing wound drainage and sloughing of grafts.

Which assessment is the nurse's highest priority in caring for a client in the acute phase of burn injury? A. Bowel sounds B. Muscle strength C. Signs of infection D. Urine output

C. Signs of infection The client with burn injury is at risk for infection as a result of open wounds and reduced immune function. Burn wound sepsis is a serious complication of burn injury, and infection is the leading cause of death during the acute phase of recovery. Assessing bowel sounds, assessing muscle strength, and assessing urine output are important but not the priority during the acute phase of burn injury.

What typical sign/symptom indicates the early stage of septic shock? A. Pallor and cool skin B. Blood pressure 84/50 mm Hg C. Tachypnea and tachycardia D. Respiratory acidosis

C. Tachypnea and tachycardia Signs of systemic inflammatory response syndrome, which precedes sepsis, include rapid respiratory rate, leukocytosis, and tachycardia. In the early stage of septic shock, the client is usually warm and febrile. Hypotension does not develop until later in septic shock due to compensatory mechanisms. Respiratory alkalosis occurs early in shock because of an increased respiratory rate.

The certified wound, ostomy, continence nurse (CWOCN) or enterostomal therapist (ET) teaches a client who has had a cystectomy about which care principles for the client's post-discharge activities? A. Nutritional and dietary care B. Respiratory care C. Stoma and pouch care D. Wiping from front to back (asepsis)

C. The enterostomal therapist demonstrates external pouch application, local skin care, pouch care, methods of adhesion, and drainage mechanisms.

A client's urinalysis shows all of the following results. Which result does the nurse report to the health care provider? A. pH 5.8 B. Osmolarity 450 C. Nitrites present D. Sodium 5 mEq/L

C. The osmolarity, pH, and sodium concentration are within normal ranges. Nitrites are not usually present in urine. Many types of bacteria, when present in the urine, convert nitrates (normally found in urine) into nitrites. A positive finding indicates a urinary tract infection.

When performing bladder scanning to detect residual urine in a female client, the nurse must first assess which of these? A. Abdominal girth B. Presence of urinary infection C. History of hysterectomy D. Hematuria

C. The scanner must be in the scan mode for male clients to ensure the scanner subtracts the volume of the uterus from the measurement.

Which assessment information about a 60-kg client admitted 12 hours ago with a full-thickness burn over 30% of the total body surface area is of greatest concern to the nurse? A. Bowel sounds are absent. B. The pulse oximetry level is 91%. C. The serum potassium level is 6.1 mEq/L. D. Urine output since admission is 370 mL.

C. The serum potassium level is 6.1 mEq/L. An elevated serum potassium level can cause cardiac dysrhythmias and arrest, and so is of the most concern. Absence of bowel sounds, a pulse oximetry level of 91%, and urine output of 370 mL since admission are normal findings during the resuscitation phase of burn injury.

The nurse corrects the nursing student when caring for a client with neutropenia secondary to chemotherapy in which circumstance? A. The student scrubs the hub of IV tubing before administering an antibiotic. B. The nurse overhears the student explaining to the client the importance of handwashing. C. The student teaches the client that symptoms of neutropenia include fatigue and weakness. D. The nurse observes the student providing oral hygiene and perineal care.

C. The student teaches the client that symptoms of neutropenia include fatigue and weakness. Symptoms of neutropenia include low neutrophil count, fever, and signs and symptoms of infection; the student should be corrected. Asepsis with IV lines is an appropriate action. Handwashing is an essential component of client care, especially when the client is at risk for neutropenia. Hygiene and perineal care help prevent infection and sepsis.

A newly admitted client who is diabetic and has pyelonephritis and prescriptions for intravenous antibiotics, blood glucose monitoring every 2 hours, and insulin administration should be cared for by which staff member? A. An RN whose other assignments include a client receiving chemotherapy for renal cell carcinoma B. An RN who is caring for a client who just returned after having renal artery balloon angioplasty C. An RN who has just completed preoperative teaching for a client who is scheduled for nephrectomy D. An RN who is currently admitting a client with acute hypertension and possible renal artery stenosis

C. This RN is caring for the most stable client and will have time to do the frequent monitoring and interventions that are needed for the newly admitted client.

The nurse is teaching the client how to provide a "clean catch" urine specimen. Which statement by the client indicates that teaching was effective? A. "I must clean with the wipes and then urinate directly into the cup." B. "I will have to drink 2 liters of fluid before providing the sample." C. "I'll start to urinate in the toilet, stop, and then urinate into the cup." D. "It is best to provide the sample while I am bathing."

C. To provide a clean catch urine sample, the client should initiate voiding, then stop, then resume voiding into the container. A midstream collection further removes secretions and bacteria because urine flushes the distal portion of the internal urethra.

The nurse is evaluating the effectiveness of fluid resuscitation for a client in the resuscitation phase of burn injury. Which finding does the nurse correlate with clinical improvement? A. Blood urea nitrogen (BUN), 36 mg/dL B. Creatinine, 2.8 mg/dL C. Urine output, 40 mL/hr D. Urine specific gravity, 1.042

C. Urine output, 40 mL/hr Fluid resuscitation is provided at the rate needed to maintain urine output at 30 to 50 mL/hr or 0.5 mL/kg/hr. A BUN of 36 mg/dL is above normal, a creatinine of 2.8 mg/dL is above normal, and a urine specific gravity of 1.042 is above normal.

The oncology nurse should use which intervention to prevent disseminated intravascular coagulation (DIC)? A. Monitoring platelets B. Administering packed red blood cells C. Using strict aseptic technique to prevent infection D. Administering low-dose heparin therapy for clients on bedrest

C. Using strict aseptic technique to prevent infection Sepsis is a major cause of DIC, especially in the oncology client. Monitoring platelets will help detect DIC, but will not prevent it. Red blood cells are used for anemia, not for bleeding/coagulation disorders. Heparin may be administered to clients with DIC who have developed clotting, but this has not been proven to prevent the disorder.

A nurse is educating a female about hygiene measures to reduce her risk for urinary tract infection. What does the nurse instruct the client to do? A. "Douche-but only once a month." B. "Use only white toilet paper." C. "Wipe from your front to your back." D. "Wipe with the softest toilet paper available."

C. Wiping front to back keeps organisms in the stool from coming close to the urethra, which increases the risk for infection.

The nurse receives report on a client with hydronephrosis. Which laboratory study should the nurse monitor? A. Hemoglobin and hematocrit (H&H) B. White blood cell (WBC) count C. Blood urea nitrogen and creatinine D. Lipid levels

C. With back pressure on the kidney, glomerular filtration is reduced or absent resulting in permanent kidney damage; BUN and creatinine are kidney function tests.

A client who has been newly diagnosed with leukemia is admitted to the hospital. Avoiding which potential problem takes priority in the client's nursing care plan? A. Fluid overload (overhydration) B. Hemorrhage C. Hypoxia D. Infection

D. Infection The main objective in caring for a newly diagnosed client with leukemia is protection from infection. Fluid overload, hemorrhage, and hypoxia are not priority problems for the client with leukemia.

An 82-year-old client with anemia is requested to receive 2 units of whole blood. Which assessment findings cause the nurse to discontinue the transfusion because it is unsafe for the client? (Select all that apply.) A. Capillary refill less than 3 seconds B. Decreased pallor C. Flattened superficial veins D. Hypertension E. Hypotension F. Rapid, bounding pulse

D, E, F In an older adult receiving a transfusion, hypertension is a sign of overload, low blood pressure is a sign of a transfusion reaction, and a rapid and bounding pulse is a sign of fluid overload. In this scenario, 2 units, or about a liter of fluid, could be problematic. Capillary refill time that is less than 3 seconds is considered to be normal and would not pose a problem. Increased (not decreased) pallor and cyanosis are signs of a transfusion reaction, while swollen (not flattened) superficial veins are present in fluid overload in older adult clients receiving transfusions.

The nurse is teaching a client who is receiving an antiestrogen drug about the side effects she may encounter. Which side effects does the nurse include in the discussion? (Select all that apply.) A. Heavy menses B. Smooth facial skin C. Hyperkalemia D. Breast tenderness E. Weight loss F. Deep vein thrombosis

D, F Breast tenderness and shrinking breast tissue may occur with antiestrogen therapy. Venous thromboembolism may also occur. Irregular menses or no menstrual period is the typical side effect of antiestrogen therapy. Acne may also develop. Hypercalcemia, not hyperkalemia, is typical. Fluid retention with weight gain may also occur.

Which laboratory test result for a client who is about to have a nephrostomy for hydronephrosis does the nurse report immediately to the physician? A. Serum sodium 137 mEq/L B. Serum potassium 4.8 mEq/L C. Blood urea nitrogen (BUN) 23 mg/dL D. International normalized ratio (INR) 4.6

D. Although the sodium and BUN levels are slightly higher than normal, they are within the ranges expected with hydronephrosis. The INR, however, is seriously elevated and indicates a dangerously long clotting time with a greatly increased risk for bleeding. It must be corrected before surgery.

The nurse is giving a group presentation on cancer prevention and recognition. Which statement by an older adult client indicates understanding of the nurse's instructions? A. "Cigarette smoking always causes lung cancer." B. "Taking multivitamins will prevent me from developing cancer." C. "If I have only one shot of whiskey a day, I probably will not develop cancer." D. "I need to report the pain going down my legs to my health care provider."

D. "I need to report the pain going down my legs to my health care provider." Pain in the back of the legs could indicate prostate cancer in an older man. Cigarette smoking is implicated in causing lung cancer and other types of cancer, but it does not always cause cancer. Investigation is ongoing about the efficacy of vitamins A and C in cancer prevention. Limiting alcohol to one drink per day is only one preventive measure.

A client with thrombocytopenia is being discharged. What information does the nurse incorporate into the teaching plan for this client? A. "Avoid large crowds." B. "Drink at least 2 liters of fluid per day." C. "Elevate your lower extremities when sitting." D. "Use a soft-bristled toothbrush."

D. "Use a soft-bristled toothbrush." Using a soft-bristled toothbrush reduces the risk for bleeding in the client with thrombocytopenia. Avoiding large crowds reduces the risk for infection, but is not specific to the client with thrombocytopenia. Increased fluid intake reduces the risk for dehydration, but is not specific to the client with thrombocytopenia. Elevating extremities reduces the risk for dependent edema, but is not specific to the client with thrombocytopenia.

A client with multiple myeloma reports bone pain that is unrelieved by analgesics. How does the nurse respond to this client's problem? A. "Ask your doctor to prescribe more medication." B. "It is too soon for additional medication to be given." C. "I'll turn on some soothing classical music for you." D. "Would you like to try some relaxation techniques?"

D. "Would you like to try some relaxation techniques?" Because most clients with multiple myeloma have local or generalized bone pain, analgesics and alternative approaches for pain management, such as relaxation techniques, are used for pain relief. This also offers the client a choice. Before prescribing additional medication, other avenues should be explored to relieve this client's pain. Even if it is too soon to give additional medication, telling that to the client is not helpful because it dismisses the client's pain concerns. Although music therapy can be helpful, this response does not give the client a choice.

A client with anemia asks, "Why am I feeling tired all the time?" How does the nurse respond? A. "How many hours are you sleeping at night?" B. "You are not getting enough iron." C. "You need to rest more when you are sick." D. "Your cells are delivering less oxygen than you need."

D. "Your cells are delivering less oxygen than you need." The single most common symptom of anemia is fatigue, which occurs because oxygen delivery to cells is less than is required to meet normal oxygen needs. Although assessment of sleep and rest is good, it does not address the cause related to the diagnosis. While it may be true that the client isn't getting enough iron, it does not relate to the client's fatigue. The statement about the client needing rest because of being sick is simply not true.

Which problem places a person at highest risk for septic shock? A. Kidney failure B. Cirrhosis C. Lung cancer D. 40% burn injury

D. 40% burn injury The skin forms the first barrier to prevent entry of organisms into the body; this client is at very high risk for sepsis and death. Although the client with kidney failure has an increased risk for infection, his skin is intact, unlike the client with burn injury. Although the liver acts as a filter for pathogens, the client with cirrhosis has intact skin, unlike the burned client. The client with lung cancer may be at risk for increased secretions and infection, but risk is not as high as for a client with open skin.

The nurse is starting the shift by making rounds. Which client does the nurse decide to assess first? A. A 42-year-old with anemia who is reporting shortness of breath when ambulating down the hallway B. A 47-year-old who recently had a Rumpel-Leede test and is requesting a nurse to "look at the bruises on my arm" C. A 52-year-old who has just had a bone marrow aspiration and is requesting pain medication D. A 59-year-old who has a nosebleed and is receiving heparin to treat a pulmonary embolism

D. A 59-year-old who has a nosebleed and is receiving heparin to treat a pulmonary embolism The client with the nosebleed may be experiencing the bleeding as a result of excessive anticoagulation and should be assessed for the severity of the situation first. The client with anemia and the client who had a Rumpel-Leede test are more stable, and can be assessed later. Making clients wait for pain medication is not desirable, but in this scenario, the client who is bleeding is the higher priority. The client waiting for pain medication should be next on the nurse's "to do" list.

The nurse has the following assignment. Which client should be encouraged to consume 2 to 3 liters of fluid each day? A. Client with chronic kidney disease B. Client with heart failure C. Client with complete bowel obstruction D. Client with hyperparathyroidism

D. A major feature of hyperparathyroidism is hypercalcemia, which predisposes to kidney stones; this client should remain hydrated.

The nurse teaches a client that intraperitoneal chemotherapy will be delivered to which part of the body? A. Veins of the legs B. Lung C. Heart D. Abdominal cavity

D. Abdominal cavity Intraperitoneal chemotherapy is placed in the peritoneal cavity or the abdominal cavity. Intravenous drugs are delivered through veins. Chemotherapy delivered into the lungs is typically placed in the pleural space (intrapleural). Chemotherapy is not typically delivered into the heart.

A 72-year-old client recovering from lung cancer surgery asks the nurse to explain how she developed cancer when she has never smoked. Which factor may explain the possible cause? A. A diagnosis of diabetes treated with insulin and diet B. An exercise regimen of jogging 3 miles four times a week C. A history of cardiac disease D. Advancing age

D. Advancing age Advancing age is the single most important risk factor for cancer. As a person ages, immune protection decreases. Diabetes is not known to cause lung cancer. Regular exercise is not a risk factor for lung cancer, nor does having cardiac disease predispose a person to lung cancer.

A client who is undergoing chemotherapy for breast cancer reports problems with concentration and memory. Which nursing intervention is indicated at this time? A. Explain that this occurs in some clients and is usually permanent. B. Inform the client that a small glass of wine may help her relax. C. Protect the client from infection. D. Allow the client an opportunity to express her feelings.

D. Allow the client an opportunity to express her feelings. Although no specific intervention for this side effect is known, therapeutic communication and listening may be helpful to the client. Evidence regarding problems with concentration and memory loss with chemotherapy is not complete, but the current thinking is that this process is usually temporary. The client should be advised to avoid the use of alcohol and recreational drugs at this time because they also impair memory. Chemotherapeutic agents are implicated in central nervous system function in this scenario, not infection.

The nurse assesses the client with which hematologic problem first? A. A 32-year-old with pernicious anemia who needs a vitamin B12 injection B. A 40-year-old with iron deficiency anemia who needs a Z-track iron injection C. A 67-year-old with acute myelocytic leukemia with petechiae on both legs D. An 81-year-old with thrombocytopenia and an increase in abdominal girth

D. An 81-year-old with thrombocytopenia and an increase in abdominal girth An increase in abdominal girth in a client with thrombocytopenia indicates possible hemorrhage; this warrants further assessment immediately. The 32-year-old with pernicious anemia, the 40-year-old with iron deficiency anemia, and the 67-year-old with acute myelocytic leukemia do not indicate any acute complications, so the nurse can assess them after assessing the client with thrombocytopenia.

A client is admitted for extracorporeal shock wave lithotripsy (ESWL). What information obtained on admission is most critical for a nurse to report to the health care provider before the ESWL procedure begins? A. "Blood in my urine has become less noticeable; maybe I don't need this procedure." B. "I have been taking cephalexin (Keflex) for an infection." C. "I previously had several ESWL procedures performed." D. "I take over-the-counter naproxen (Aleve) twice a day for joint pain."

D. Because a high risk for bleeding during ESWL has been noted, clients should not take NSAIDs before this procedure. The ESWL will have to be rescheduled for this client.

When planning an assessment of the urethra, what does the nurse do first? A. Examines the meatus B. Notes any unusual discharge C. Records the presence of abnormalities D. Dons gloves

D. Before examination begins, body fluid precautions (gloves) must be donned first.

A client has a glioblastoma. The nurse begins to plan care for this client with which type of cancer? A. Liver B. Smooth muscle C. Fatty tissue D. Brain

D. Brain The prefix "glio-" is used when cancers of the brain are named. The prefix "hepato-" is included when cancers of the liver are named. The prefix "leiomyo-" is included when cancers of smooth muscle are named. The prefix "lipo-" is included when cancers of fat or adipose tissue are named.

A client with hypovolemic shock has these vital signs: temperature 97.9° F; pulse 122 beats/min; blood pressure 86/48 mm Hg; respirations 24 breaths/min; urine output 20 mL for last 2 hours; skin cool and clammy. Which medication order for this client does the nurse question? A. Dopamine (Intropin) 12 mcg/kg/min B. Dobutamine (Dobutrex) 5 mcg/kg/min C. Plasmanate 1 unit D. Bumetanide (Bumex) 1 mg IV

D. Bumetanide (Bumex) 1 mg IV A diuretic such as bumetanide will decrease blood volume in a client who is already hypovolemic; this order should be questioned because this is not an appropriate action to expand the client's blood volume. The other orders are appropriate for improving blood pressure in shock, and do not need to be questioned.

Which of these staff members should be assigned to a client who has benign prostatic hyperplasia and hydronephrosis and needs an indwelling catheter inserted? A. An RN float nurse who has 10 years of experience with pediatric clients B. An LPN/LVN who has worked in the hospital's kidney dialysis unit until recently C. An RN without recent experience who has just completed an RN refresher course D. An LPN/LVN with 5 years of experience in an outpatient urology surgery center

D. Catheterization of a client with an enlarged prostate, a skill within the scope of practice of the LPN/LVN, would be performed frequently in a urology center.

When caring for an obtunded client admitted with shock of unknown origin, which action does the nurse take first? A. Obtain IV access and hang prescribed fluid infusions. B. Apply the automatic blood pressure cuff. C. Assess level of consciousness and pupil reaction to light. D. Check the airway and respiratory status.

D. Check the airway and respiratory status. When caring for any client, determining airway and respiratory status is the priority. The airway takes priority over obtaining IV access, applying the blood pressure cuff, and assessing for changes in the client's mental status.

When delegating care for clients on the burn unit, which client does the charge nurse assign to an RN who has floated to the burn unit from the intensive care unit? A. Burn unit client who is being discharged after 6 weeks and needs teaching about wound care B. Recently admitted client with a high-voltage electrical burn C. A client who has a 25% total body surface area (TBSA) burn injury, for whom daily wound débridement has been prescribed D. Client receiving IV lactated Ringer's solution at 150 mL/hr

D. Client receiving IV lactated Ringer's solution at 150 mL/hr An RN float nurse will be familiar with administration of IV fluids and with signs of fluid overload, such as shortness of breath, and so could be assigned to the client receiving IV lactated Ringer's solution at 150 mL/hr. The client needing teaching about wound care, the client with a high-voltage electrical burn, and the client with a 25% TBSA burn injury all require specialized knowledge about burn injuries and should be assigned to RNs who have experience caring for clients with burn injuries.

The nurse presents a cancer prevention program to teens. Which instruction will have the greatest impact in cancer prevention? A. Avoid asbestos. B. Wear sunscreen. C. Get the human papilloma virus (HPV) vaccine. D. Do not smoke cigarettes.

D. Do not smoke cigarettes. All of these actions are part of cancer prevention; however, tobacco is the single most important source of preventable carcinogenesis. Asbestos may be found in older homes and buildings. Most schools have been through an asbestos abatement program so should not pose a risk. It would be important to share with teens who may be involved in the construction industry during the summer to be aware of asbestos risks. Although asbestos may present a risk for lung cancer, it is not a likely exposure for teens. Lifetime exposure to the sun and the use of tanning beds will increase the risk for cancer, but not as much as tobacco use. The HPV vaccine will decrease the risk for cervical cancer, but will not have as much of an impact on cancer prevention as avoiding tobacco.

What does the nurse teach the client to prevent the risk for urinary tract infection (UTI)? A. Limit fluid intake. B. Increase caffeine consumption. C. Limit sugar intake. D. Drink about 3 liters of fluid daily.

D. Drinking about 3 liters of fluid daily, if another medical problem does not require fluid restriction, helps prevent dehydration and UTIs.

A nurse is caring for clients on a renal/kidney medical-surgical unit. Which drug, requested by a health care provider, for a client with a urinary tract infection (UTI) does the nurse question? A. Bactrim B. Cipro C. Noroxin D. Tegretol

D. Drug alerts state that confusion is frequent (sound alike and look alike) between the drugs Tequin and Tegretol. The former is used for UTI, and the latter is prescribed as an oral anticonvulsant.

The client had IV urography 8 hours ago. Which nursing intervention is the priority for this client? A. Maintaining bedrest B. Medicating for pain C. Monitoring for hematuria D. Promoting fluid intake

D. Ensure adequate hydration by urging the client to take oral fluid or by giving IV fluids. Hydration reduces the risk for kidney damage.

A nurse is teaching a client who is scheduled for a neobladder and a Kock pouch. Which client statement indicates correct understanding of these procedures? A."If I restrict my oral intake of fluids, the adjustment will be easier." B. "I must go to the restroom more often because my urine will be excreted through my anus." C. "I need to wear loose-fitting pants so the urine can flow into my ostomy bag." D. "I will have to drain my pouch with a catheter."

D. For the client with a neobladder and a Kock pouch, urine is collected in a pouch and is drained with the use of a catheter.

The nurse is assessing the nutritional status of a client with anemia. How does the nurse obtain information about the client's diet? A. Asks the client to rate his or her diet on a scale of 1 (poor) to 10 (excellent) B. Determines who prepares the client's meals and plans an interview with him or her C. From a prepared list, finds out the client's food preferences D. Has the client write down everything he or she has eaten for the past week

D. Has the client write down everything he or she has eaten for the past week Having the client provide a list of items eaten in the past week is the most accurate way to find out what the client likes and dislikes, as well as what the client has been eating. It will provide information about "junk" food intake, as well as protein, vitamin, and mineral intake. Rating scales are good for subjective data collection about some conditions such as pain, but the subjectivity of a response such as this does not provide the nurse with specific data needed to assess a diet. Interviewing the food preparer is time-consuming and poses several problems, such as whether a number of people are preparing meals, or if the client goes "out" for meals. Determining food preferences from a prepared list provides information about what the client enjoys eating, not necessarily what the client has been eating; for instance, the client may like steak but may be unable to afford it.

Which client does the nurse manager on a medical unit assign to an experienced LPN/LVN? A. 42-year-old with painless hematuria who needs an admission assessment B. 46-year-old scheduled for cystectomy who needs help in selecting a stoma site C. 48-year-old receiving intravesical chemotherapy for bladder cancer D. 55-year-old with incontinence who has intermittent catheterization prescribed

D. Insertion of catheters is within the education and legal scope of practice for LPNs/LVNs.

For which client living at home is intermittent self-catheterization an inappropriate method for incontinence management? A. 36-year-old woman who is blind B. 46-year-old man who has paraplegia C. 56-year-old woman who has diabetes D. 66-year-old man who has severe osteoarthritis

D. Intermittent self-catheterization requires significant manual dexterity to reach the area and perform the catheterization without contaminating the catheter. A person with severe osteoarthritis is not likely to be able to perform this technique safely. The technique does not require vision or the use of the lower extremities. A person with diabetes would be at increased risk for infection and would need to understand how to avoid contamination.

Which type of cancer has been associated with Down syndrome? A. Breast cancer B. Colorectal cancer C. Malignant melanoma D. Leukemia

D. Leukemia Leukemia is associated with Down syndrome and Turner syndrome. Breast cancer is often found clustered in families, not in association with Down syndrome. Colorectal cancer is associated with familial polyposis. Malignant melanoma is associated with familial clustering and sun exposure.

Which statement about the process of malignant transformation is correct? A. Mutation of genes is an irreversible event that always leads to cancer development in the initiation phase. B. Insulin and estrogen enhance the division of an initiated cell during the promotion phase. C. Tumors form when carcinogens invade the gene structure of the cell in the latency phase. D. Nutrition of cancer cells is provided by tumor angiogenesis factor (TAF) in the promotion stage.

D. Nutrition of cancer cells is provided by tumor angiogenesis factor (TAF) in the promotion stage. The promotion phase consists of progression when the blood supply changes from diffusion to TAF. Insulin and estrogen increase cell division. If cell division is halted, mutation of genes does not lead to cancer development in the initiation phase. In the initiation phase, carcinogens invade the DNA of the nucleus of a single cell. A 1-cm tumor consists of 1 billion cells. The latency phase occurs between initiation and tumor formation.

Which would be an appropriate task to delegate to unlicensed assistive personnel (UAP) working on a medical-surgical unit? A. Administering erythropoietin to a client with myelodysplastic syndrome B. Assessing skin integrity on an anemic client who fell during ambulation C. Assisting a client with folic acid deficiency in making diet choices D. Obtaining vital signs on a client receiving a blood transfusion

D. Obtaining vital signs on a client receiving a blood transfusion Obtaining vital signs on a client is within the scope of practice for UAP. Administering medication, assessing clients, and assisting with prescribed diet choices are complex actions that should be done by licensed nurses.

The home health nurse is caring for a client who has a history of a kidney transplant and takes cyclosporine (Sandimmune) and prednisone (Deltasone) to prevent rejection. Which assessment finding is most important to communicate to the transplant team? A. Temperature of 96.6° F B. Reports of joint pain C. Pink and dry oral mucosa D. Palpable lump in the client's axilla

D. Palpable lump in the client's axilla Clients taking immunosuppressive drugs to prevent rejection are at increased risk for the development of cancer; any lump should be reported to the physician. Fever should be reported to the physician, but this client's temperature is normal. It is not necessary to report joint pain to the transplant team; it is not a sign of rejection and is not a complication of transplant. A pink and dry oral mucosa may be a sign of dehydration, but it is not necessary to report this to the transplant team.

A health care provider requests phenazopyridine (Pyridium) for a client with cystitis. What does the nurse tell the client about the drug? A. "It will act as an antibacterial drug." B. "This drug will treat your infection, not the symptoms of it." C. "You need to take the drug on an empty stomach." D. "Your urine will turn red or orange while on the drug."

D. Phenazopyridine (Pyridium) will turn the client's urine red or orange. Care should be taken because it will stain undergarments. Clients should be warned about this effect of the drug because it will be alarming to them if they are not informed.

A client is in the acute phase of burn injury. For which action does the nurse decide to coordinate with the registered dietitian? A. Discouraging having food brought in from the client's favorite restaurant B. Providing more palatable choices for the client C. Helping the client lose weight D. Planning additions to the standard nutritional pattern

D. Planning additions to the standard nutritional pattern Nutritional requirements for the client with a large burn area can exceed 5000 kcal/day. In addition to a high calorie intake, the burn client requires a diet high in protein for wound healing. Consultation with the dietitian is required to help the client achieve the correct nutritional balance. It is fine for the client with a burn injury to have food brought in from the outside. The hospital kitchen can be consulted to see what other food options may be available to the client. It is not therapeutic for the client with burn injury to lose weight.

A client who is 6 months pregnant comes to the Prenatal Clinic with a suspected urinary tract infection (UTI). What action does the nurse take with this client? A. Discharges the client to her home for strict bedrest for the duration of the pregnancy B. Instructs the client to drink a minimum of 3 liters of fluids, especially water, every day to "flush out" bacteria C. Recommends that the client refrain from having sexual intercourse until after she has delivered her baby D. Refers the client to the Clinic Nurse Practitioner (CNP) for immediate follow-up

D. Pregnant women with UTI require prompt and aggressive treatment because simple cystitis can lead to acute pyelonephritis. This in turn can cause preterm labor-with adverse effects for the fetus.

The nurse is caring for a client who is in sickle cell crisis. What action does the nurse perform first? A. Apply cool compresses to the client's forehead. B. Encourage the client's use of two methods of birth control. C. Increase food sources of iron in the client's diet. D. Provide pain medications as needed.

D. Provide pain medications as needed. Analgesics are needed to treat sickle cell pain. Warm soaks or compresses can help reduce pain perception. Cool compresses do not help the client in sickle cell crisis. Birth control is not the priority for this client. Increasing iron in the diet is not pertinent for the client in sickle cell crisis.

An outpatient client is receiving photodynamic therapy. Which environmental factor is a priority for the client to adjust for protection? A. Storing drugs in dark locations at room temperature B. Wearing soft clothing C. Wearing a hat and sunglasses when going outside D. Reducing all direct and indirect sources of light

D. Reducing all direct and indirect sources of light Lighting of all types must be kept to a minimum with clients receiving photodynamic therapy; it can lead to burns of the skin and damage to the eyes because these clients' eyes are sensitive to light. Any drug that the client is prescribed should be considered for its photosensitivity properties; drugs should be stored according to the recommendations, but this is not the primary concern for this client. Clothing must cover the skin to prevent burns from direct or indirect light; texture is not a concern for the client receiving this treatment. The client will be homebound for 1 to 3 months after the treatment and should not go outside.

Which action does the nurse delegate to unlicensed assistive personnel (UAP)? A. Drawing a partial thromboplastin time from a saline lock on a client with a pulmonary embolism B. Performing a capillary fragility test to check vascular hemostatic function on a client with liver failure C. Referring a client with a daily alcohol consumption of 12 beers for counseling D. Reporting any bleeding noted when catheter care is given to a client with a history of hemophilia

D. Reporting any bleeding noted when catheter care is given to a client with a history of hemophilia Reporting findings during routine care is expected and required of unlicensed staff members. Drawing a partial thromboplastin time, performing a capillary fragility test, and referring a client for alcohol counseling are more complex and should be done by licensed nursing staff.

What is the most important environmental risk for developing leukemia? A. Direct contact with others with leukemia B. Family history C. Living near high-voltage power lines D. Smoking cigarettes

D. Smoking cigarettes According to the American Cancer Society (ACS), the only proven lifestyle-related risk factor for leukemia is cigarette smoking. Leukemia is not contagious. Genetics is a strong indicator, but it is not an environmental risk factor. According to the ACS, living near high-voltage power lines is not a proven risk factor for leukemia.

A client who is receiving a blood transfusion suddenly exclaims to the nurse, "I don't feel right!" What does the nurse do next? A. Call the Rapid Response Team. B. Obtain vital signs and continue to monitor. C. Slow the infusion rate of the transfusion. D. Stop the transfusion.

D. Stop the transfusion. The client may be experiencing a transfusion reaction; the nurse should stop the transfusion immediately. Calling the Rapid Response Team or obtaining vital signs is not the first thing that should be done. The nurse should not slow the infusion rate, but should stop it altogether.

Which nursing intervention or practice is most effective in helping to prevent urinary tract infection (UTI) in hospitalized clients? A. Encouraging them to drink fluids B. Irrigating all catheters daily with sterile saline C. Recommending catheters should be placed in all clients D. Re-evaluating periodically the need for indwelling catheters

D. Studies have shown that re-evaluating the need for indwelling catheters in clients is the most effective way to prevent UTI in the hospital setting.

Which nurse should be assigned to care for an intubated client who has septic shock as the result of a methicillin-resistant Staphylococcus aureus (MRSA) infection? A. The LPN/LVN who has 20 years of experience B. The new RN who recently finished orienting and is working independently with moderately complex clients C. The RN who will also be caring for a client who had coronary artery bypass graft (CABG) surgery 12 hours ago D. The RN with 2 years of experience in intensive care

D. The RN with 2 years of experience in intensive care The RN with current intensive care experience who is not caring for a postoperative client would be an appropriate assignment. Care of the unstable client with intubation and mechanical ventilation is not within the scope of practice for the LPN/LVN. A client who is experiencing septic shock is too complex for the new RN. Although the RN who is also caring for the post-CABG client is experienced, this assignment will put the post-CABG client at risk for MRSA infection.

The client is scheduled for a cystoscopy later this morning. The consent form is not signed, and the client has not had any preoperative medication. The nurse notes that the provider visited the client the day before. What action does the nurse take? A. Asks the client to sign the informed consent B. Cancels the procedure C. Asks the client's spouse to sign the form D. Notifies the department and the provider

D. The client may be asked to sign the consent form in the department; notifying both the provider and the department ensures communication across the continuum of care, with less likelihood of omission of information.

A nurse is talking to adult clients about urinary and sexual hygiene. Which words does the nurse use when referring to the client's reproductive body parts? A. Children's terms that are easily understood B. Slang words and terms that are heard "socially" C. Technical and medical terminology D. Words that the client uses

D. The nurse should use the terms with which the client is most familiar, so there is no chance for the client to misunderstand information. Using the client's language ensures the comfort level for the client.

The client with diabetes who also has persistent proteinuria asks what he could do to prevent eventual kidney failure. What is the nurse's best response? A. "Wear pads and other protective gear around your lower back when engaging in contact sports." B. "Drink at least 3 L of water daily and avoid carbonated beverages." C. "Limit your intake of proteins to less than 100 g daily." D. "Keep your blood glucose levels in the target range."

D. The presence of persistent proteinuria indicates kidney damage has already occurred and is likely to eventually progress to kidney failure. Although kidney failure cannot be prevented, the decline in kidney function can be slowed with tight glycemic control. The severity of diabetic kidney disease is related to the degree of hyperglycemia the client generally experiences. With poor control of hyperglycemia, the complicating problems of atherosclerosis, hypertension, and neuropathy (which promote loss of bladder tone, urinary stasis, and urinary tract infection) are more severe and more likely to cause additional kidney damage sooner.

Which technique does the nurse use to obtain a sterile urine specimen from the client with a Foley catheter? A. Disconnects the Foley catheter from the drainage tube and collects urine directly from the Foley B. Removes the existing catheter and obtains a sample during the process of inserting a new Foley C. Uses a sterile syringe to withdraw urine from the urine collection bag D. Clamps the tubing, attaches a syringe to the specimen, and withdraws at least 5 mL of urine

D. This is the correct technique for obtaining a sterile urine specimen from the client with a Foley catheter.

An older adult woman confides to a nurse, "I am so embarrassed about buying adult diapers for myself." How does the nurse respond? A. "Don't worry about it. You need them." B. "Shop at night-when stores are less crowded." C. "Tell everyone that they are for your husband." D. "That is tough. What do you think might help?"

D. This response acknowledges the client's concerns and attempts to help the client think of methods to solve her problem.

The client is a burn victim who is noted to have increasing edema and decreased urine output as a result of the inflammatory compensation response. What does the nurse do first? A. Administer a diuretic. B. Provide a fluid bolus. C. Recalculate fluid replacement based on time of hospital arrival. D. Titrate fluid replacement.

D. Titrate fluid replacement. The intravenous fluid rate should be adjusted on the basis of urine output plus serum electrolyte values (titration of fluids). A common mistake in treatment is giving diuretics to increase urine output. Giving a diuretic will actually decrease circulating volume and cardiac output by pulling fluid from the circulating blood volume to enhance diuresis. Fluid boluses are avoided because they increase capillary pressure and worsen edema. Fluid replacement formulas are calculated from the time of injury, not from the time of arrival at the hospital.

The client with pyelonephritis has been prescribed urinary antiseptic medication. What purpose does this medication serve? A. Decreases bacterial count B. Destroys white blood cells C. Enhances the action of antibiotics D. Provides comfort

D. Urinary antiseptic drugs such as nitrofurantoin (Macrodantin) are prescribed to provide comfort for clients with pyelonephritis.

A client is referred to a home health agency after being hospitalized with overflow incontinence and a urinary tract infection (UTI). Which nursing action can the home health RN delegate to a home health aide? A. Assisting the client in developing a schedule for when to take prescribed antibiotics B. Inserting a straight catheter as necessary if the client is unable to empty the bladder C. Teaching the client how to use the Credé maneuver to empty the bladder more fully D. Using a bladder scanner (with training) to check residual bladder volume after the client voids

D. Use of a bladder scanner is noninvasive and can be accomplished by a home health aide (UAP) who has been trained and evaluated in this skill.

A client is scheduled for a bone marrow aspiration. What does the nurse do before taking the client to the treatment room for the biopsy? A. Clean the biopsy site with an antiseptic or povidone-iodine (Betadine). B. Hold the client's hand and ask about concerns. C. Review the client's platelet (thrombocyte) count. D. Verify that the client has given informed consent.

D. Verify that the client has given informed consent. Verifying informed consent must be done before the procedure can be performed. A signed permit must be on the client's chart. Cleaning the biopsy site is done before the procedure, but this is not done until consent is verified; it will be done just before the procedure is performed. Holding the client's hand and offering verbal support may be done during the procedure, but the procedure cannot be completed until the consent is signed. Reviewing the client's platelet count is not imperative.

The nurse is mentoring a recent graduate RN about administering blood and blood products. What does the nurse include in the data? A. Obtain the client's initial set of vital signs (VS) within the first 10 minutes of the infusion. B. Remain with the client who is receiving the blood for the first 5 minutes of the infusion. C. Use a 22-gauge needle to obtain venous access when starting the infusion. D. Verify with another RN all of the data on blood products.

D. Verify with another RN all of the data on blood products. All data are checked by two RNs. Human error is the most common cause of ABO incompatibilities in administering blood and blood products. Initial VS should be recorded before the start of infusion of blood, not after it has begun. The nurse remains with the client for the first 15 to 30 minutes (not 5) of the infusion. This is the period when any transfusion reactions are likely to happen. A 20-gauge needle (or a central line catheter) is used; the 22-gauge needle is too small.

The nurse is transfusing a unit of whole blood to a client when the health care provider requests the following: "Furosemide (Lasix) 20 mg IV push." What does the nurse do? A. Add furosemide to the normal saline that is infusing with the blood. B. Administer furosemide to the client intramuscularly (IM). C. Piggyback furosemide into the infusing blood. D. Wait until the transfusion has been completed to administer furosemide.

D. Wait until the transfusion has been completed to administer furosemide. Completing the transfusion before administering furosemide is the best course of action in this scenario. Drugs are not to be administered with infusing blood products; they can interact with the blood, causing risks for the client. Stopping the infusing blood to administer the drug and then restarting it is also not the best decision. Changing the admission route is not a nursing decision.

The nurse is caring for a client with neutropenia. Which clinical manifestation indicates that an infection is present or should be ruled out? A. Coughing and deep breathing B. Evidence of pus C. Fever of 102° F or higher D. Wheezes or crackles

D. Wheezes or crackles Wheezes or crackles in the neutropenic client may be the first symptom of infection in the lungs. Coughing and deep breathing are not indications of infection, but can help prevent it. The client with leukopenia, not neutropenia, may have a severe infection without pus or with only a low-grade fever.

A nurse is teaching a client about pelvic muscle exercises. What information does the nurse include? A. "For the best effect, perform all your exercises while you are seated on the toilet." B. "Limit your exercises to 5 minutes twice a day, or you will injure yourself." C. "Results should be visible to you within 72 hours." D. "You know that you are exercising correct muscles if you can stop urine flow in midstream."

D. When the client can start and stop the urine stream, the pelvic muscles are being used.

The nurse is caring for postoperative clients at risk for hypovolemic shock. Which condition represents an early symptom of shock? A. Hypotension B. Bradypnea C. Heart blocks D. Tachycardia

D. tachycardia Heart and respiratory rates increased from the client's baseline level or a slight increase in diastolic blood pressure may be the only objective manifestation of this early stage of shock. Catecholamine release occurs early in shock as a compensation for fluid loss; blood pressure will be normal. Early in shock, the client displays rapid, not slow, respirations. Dysrhythmias are a late sign of shock; they are related to lack of oxygen to the heart.


संबंधित स्टडी सेट्स

The Cell-an Introduction: Biology

View Set

Intro to Data Science and AI Maastricht Lectures 1-6b

View Set

SECURITY + PENETRATION TESTING 6.13

View Set

Practice Question Banks 16-30 (Not Required)

View Set

Certified Ethical Hacking (CEH) v.8 Study Guide part 2 (101-200)

View Set

Gaston College NUR 112 Appendicitis

View Set

International Marketing Final Exam Review, TTU, Duhan

View Set